sbmptn revisi

55
SOAL- SOAL SBMPTN I. EJAAN YANG DISEMPURNAKAN DAN BENTUK KATA 1. …. penerapan kebijakan itu menyebabkan munculnya rasa tidak puas … para pendukung setia bupati itu Kata baku yang tepat untuk melengkapi kalimat tersebut adalah…. a. Ketidak-konsisitenan, di antara d. Ketidakkonsistenan, di antara b. Ketidak konsistenan, di antara e. Ketidak-konsistenan, diantara c. Ketidakkonsistenan, diantara 2. Penulisan kata yang sesuai dengan EyD terdapat pada kalimat…. a. Ia akan mempertanggungjawabkan perbuatannya. b. Sudah saya beritahukan masalah itu kepadanya. c. Kami tidak bertanggungjawab atas peristiwa itu. d. Tolong bertitahu mereka! e. Pemberi-tahuan itu datang terlambat. 3. Penggunaan kata serapan yang tepat terdapat pada kalimat….. a. Dari askpek bahasa, kata kampanye berasal dari bahasa Inggris, yaitu to campaign yang berarti ‘serangkaian aktivitas yang direncanakan atau suatu operasi militer’. b. Sebagai penyusun buku ini kamiberharap buku ini dapat menggugah generasi muda perempuan bahwa dalam berkeluarga membina karir masing-masing adalah suatu kemungkinan. c. Dengan menerbitkan buku ini kami ingin menyumbangkan sesuatu yang kongkrit tentang aspirasi perempuan Indonesia sebagai manusia dan anggota masyarakat yang sedang membangun. d. Penulis beranggapan bahwa praktek perladangan, sebagai contoh, merupakan suatu keahlian yang didasari pengalaman tradisional dan sama sekali tidak primitif. e. Makin banyak lulusan sekolah menengah atas yang berminat mendalami bidang studi akutansi. 4. Penulisan huruf kapital yang tepat terdapat pada kalimat…. a. Suasana sore di Selat Bali sangat indah. b. Lulusan SMA itu berminat menjadi Sekretaris di perusahaan. c. Menguasai Bahasa Indonesia berarti mencintai Negara Indonesia. d. Paling enak menikmati tahu Sumedang di pinggir danau. e. Besok Gubernur itu akan mersemikan taman kota di daerah Menteng. 5. Pembentukan kata berikut yang tidak sesuai dengan EyD adalah…. a. Mengkliring, mencap, mengkop d. mengelap, mengkhitan, mengilap b. Mengklakson, mengeset, mengicau e. mengklarifikasi, mengklise, mengontrol c. Mengeklik, mengelem, mengelas 6. Penggunaan ejaan yang tidak tepat terdapat pada kalimat…. a. Selain itu mereka juga menerima tantangan berusaha yang penuh resiko. b. Bahan tertulis yang diberikan umumnya berupa fotokopi bahan presentasi. c. Pada 2010/ 2011 prosentase anggaran pendidikan terhadap PDB di Indonsia tergolong paling kecil. d. Diharapkan kegiatan ini dapat menimbulkan dampak positif terhadap perkembangan dusun dalam jenjang hierarki pemerintahan desa. e. Setiap mau pergi anak itu selalu minta izin dahulu kepada kedua orang tuanya. 7. Penulisan kata gabung yang tidak tepat terdapat pada jawaban…. a. Tanggung jawab, kerjasama, tandatangan, terima kasih b. Semipermanen, mancanegara, pascasarjana, saptakrida c. Acapkali, adakalanya, barangkali, bagaimana d. Mata ajar, meja tulis, orang tua, simpang empat e. Kacamata, dukacita, satu tangan, halabihalal 8. Penulisan tanda baca titik yang sesuai dengan EyD adalah…. a. Rp 1.000.000,00 d. Rp. 1.000.000,00

Upload: rizki-akbar

Post on 14-Jul-2016

303 views

Category:

Documents


13 download

DESCRIPTION

latihan soal sbmptn bahasa indonesia

TRANSCRIPT

Page 1: SBMPTN REVISI

SOAL- SOAL SBMPTN I. EJAAN YANG DISEMPURNAKAN DAN BENTUK KATA

1. …. penerapan kebijakan itu menyebabkan munculnya rasa tidak puas … para pendukung setia bupati itu Kata baku yang tepat untuk melengkapi kalimat tersebut adalah…. a. Ketidak-konsisitenan, di antara d. Ketidakkonsistenan, di antara b. Ketidak konsistenan, di antara e. Ketidak-konsistenan, diantara c. Ketidakkonsistenan, diantara

2. Penulisan kata yang sesuai dengan EyD terdapat pada kalimat…. a. Ia akan mempertanggungjawabkan perbuatannya. b. Sudah saya beritahukan masalah itu kepadanya. c. Kami tidak bertanggungjawab atas peristiwa itu. d. Tolong bertitahu mereka!e. Pemberi-tahuan itu datang terlambat.

3. Penggunaan kata serapan yang tepat terdapat pada kalimat….. a. Dari askpek bahasa, kata kampanye berasal dari bahasa Inggris, yaitu to campaign yang berarti ‘serangkaian

aktivitas yang direncanakan atau suatu operasi militer’. b. Sebagai penyusun buku ini kamiberharap buku ini dapat menggugah generasi muda perempuan bahwa dalam

berkeluarga membina karir masing-masing adalah suatu kemungkinan. c. Dengan menerbitkan buku ini kami ingin menyumbangkan sesuatu yang kongkrit tentang aspirasi perempuan

Indonesia sebagai manusia dan anggota masyarakat yang sedang membangun. d. Penulis beranggapan bahwa praktek perladangan, sebagai contoh, merupakan suatu keahlian yang didasari

pengalaman tradisional dan sama sekali tidak primitif. e. Makin banyak lulusan sekolah menengah atas yang berminat mendalami bidang studi akutansi.

4. Penulisan huruf kapital yang tepat terdapat pada kalimat…. a. Suasana sore di Selat Bali sangat indah. b. Lulusan SMA itu berminat menjadi Sekretaris di perusahaan. c. Menguasai Bahasa Indonesia berarti mencintai Negara Indonesia. d. Paling enak menikmati tahu Sumedang di pinggir danau. e. Besok Gubernur itu akan mersemikan taman kota di daerah Menteng.

5. Pembentukan kata berikut yang tidak sesuai dengan EyD adalah…. a. Mengkliring, mencap, mengkop d. mengelap, mengkhitan, mengilapb. Mengklakson, mengeset, mengicau e. mengklarifikasi, mengklise, mengontrolc. Mengeklik, mengelem, mengelas

6. Penggunaan ejaan yang tidak tepat terdapat pada kalimat…. a. Selain itu mereka juga menerima tantangan berusaha yang penuh resiko. b. Bahan tertulis yang diberikan umumnya berupa fotokopi bahan presentasi. c. Pada 2010/ 2011 prosentase anggaran pendidikan terhadap PDB di Indonsia tergolong paling kecil. d. Diharapkan kegiatan ini dapat menimbulkan dampak positif terhadap perkembangan dusun dalam jenjang

hierarki pemerintahan desa. e. Setiap mau pergi anak itu selalu minta izin dahulu kepada kedua orang tuanya.

7. Penulisan kata gabung yang tidak tepat terdapat pada jawaban…. a. Tanggung jawab, kerjasama, tandatangan, terima kasih b. Semipermanen, mancanegara, pascasarjana, saptakrida c. Acapkali, adakalanya, barangkali, bagaimanad. Mata ajar, meja tulis, orang tua, simpang empat e. Kacamata, dukacita, satu tangan, halabihalal

8. Penulisan tanda baca titik yang sesuai dengan EyD adalah…. a. Rp 1.000.000,00 d. Rp. 1.000.000,00 b. Rp 1.000.000,- e. Rp1.000.000,00 c. Rp. 1.000.000,-

9. Penulisan kata yang semuanya benar terdapat pada jawaban…. a. Kuitansi, kwalitas, jadual d. kwitansi, kualitas, jadwal b. Kuitansi, kualitas, jadwal e. kwitansi, kwalitas, jadwal c. Kuitansi, kualitas, jadual

10. Penulisan kata gabung yang tidak sesuai dengan penulisan kata baku adalah….a. Mitrabestari, matahari, tatabahasa d. halalbihalal, acapkali, citarasab. Bilamana, saputangan, darmabakti e. sukacita, sukaria, tunarungu c. Peribahasa, sumber daya, syhbandar

11. Dalam suatu penelitian, analisis data dapat dilakukan setelah data terkumpul. Namun, dalam penelitian kualitaif, analisis data dapat dilakukan berbarengan dengan proses mengumpulkan data. Dalam proses mengumpulkan data ini, penelitian melakukan penyeleksian, pengklasifikasian, penafsiran, pengkategorian, dan penjelasan data. Bentukan kata yang tidak sesuai dalam teks tersebut adalah…. a. berbarengan d. pengkategorian b. penyeleksian e. penafsiran c. pengklasifikasin

Page 2: SBMPTN REVISI

12. Ahli kesehatan menyarankan agar para penderita osteoporosis banyak mengonsumsi sari kedelai karena dalam sari kedelai ini mengandung kalsium yang baik untuk mencegah terjadinya osteoporosis. Sari kedelai ini dapat dikonsumsi oleh siapa saja dengan risiko yang rendah. Selain itu, sari kedelai yang terkemas dalam bentuk minuman segar dapat diperoleh dengan mudah di warung-warung makanan. Bentukan kata yang tidak tepat pada kutipan tersebut adalah…. a. Menyarankan d. mencegahb. Mengonsumsi e. terkemas c. Mengandung

13. Pemakaian konfiks yang benar pada gabungan kata terdapat pada…. a. Pertangggungan jawab d. pendayaan guna b. Keikutsertaan e. dibebaskan tugas c. Pengalihan bahasa

14. Penulisan kata gabung di bawah ini benar, kecuali…. a. Berlipat ganda d. putar balikkan b. Garis bawahi e. menduakalikan c. Memberi tahukan

15. …permainan golf membuat Indonesia juga kecipratan rezeki. Bermunculanlah industri menengah dan kecil yang menyediakan…golf ini cukup laris di …lokal karena selain harganya lebih murah daripada barang impor, kulaitasnya juga mulai dapat…. Kata baku yang tepat untuk mengisi bagian rumpang pada teks tersebut adalah… a. Meningkatnya, aksesoris, perdagangan, diunggulkan b. Menguatnya, aksesori, perdagangan, dipercayac. Mendunianya, aksesori, pasaran, diandalkan d. Memopulernya, aksesoris, pasaran, diandalkan e. Meluasnya, aksesori, pasaran, diutamakan

16. Berawal dari dua ekor, kini jalak bali milik Soehana bertambah banyak. Berhubung untuk memilikinya harus ada izin dari Balai Konservasi Daya Alam (BKSDA), Soehana pun rela …birokrasi untuk memperoleh izin … sekaligus izin … Lebih dari setahun ia mengurus izin itu. hasilnya adalah kedua izin yang disertai dengan sertifikat sehingga bagi para pehobi yang ingin mengikuti jejaknya bisa memperoleh indukan yang sah dari Soehana. Hingga kini sudah ada seratus sepuluh jalak bali didaftarkan di BKSDA. Kata baku yang tepat untuk mengisi bagian yang rumpang pada paragraf tersebut adalah… a. Melobi, pemelihara, pembiakan b. Menembus, kepemilikan, penangkaran c. Menempuh, pemeliharaan, pengembangbiakan d. Memninta izin, penangkaran, pengembangan e. Menerobos, pemilik, pembudidayaan

17. Pelaku penembakan itu kata henry sudah pasti merupakan bagian dari jaringan narkoba. Waktu itu, sekitar 5 bulan lalu, mereka berkejar-kejaran di jalan. Dua mobil menggencetnya di jalan tol. Kata dan tanda baca pada kalimat tersebut tidak baku. Beberapa cara berikut dapat dipakai untuk memperbaiki, kecuali…. a. Menambahkan tanda baca koma(,) sesudah kata penembakan itu b. Mengganti kata berkejar-kejaran dengan saling mengejar c. Menambahkan tanda baca koma (,) setelah kata Henryd. Mengganti kata jaringan dengan sindikate. Menuliskan angka 5 dengan kata lima

18. Penggunaan huruf kapital yang tidak tepat terdapat pada…. a. Ajax telah menghancurkan Real Madrid. b. Piala Toyota memunyai nilai strategis. c. Juara Liga Korea Selatan mengalahkan Persib Bandung. d. Juara bertahan Liga Belanda, Ajax, tampak di arena piala Toyota. e. Diego Maradona adalah bintang lapangan yang tangguh.

19. Penggunaan huruf kapital yang benar terdapat pada pilihan jawaban….a. Kita berusaha menggunakan Bahasa indonesia yang baik dan benar. b. Pada Bulan Agustus ia akan berangkat ke Amerika. c. Di mana banyak terdapat Suku Jawa? d. Pegunungan yang membentang di dataran Sumatera itu bernama Bukit Barisan. e. Dengan gembira masyarakat menyambut Hari Lebaran.

20. Penulisan lambang bilangan yang benar terdapat pada pilihan jawaban…. a. 25 orang yang hadir pada acara itu.b. Kami membutuhkan 15 (lima belas) pegawai baru. c. Untuk keperluan itu kami harus menyiapkan uang 1. 000 an. d. Tiga ratus peserta yang hadir pada kegiatan sosialisasi itu. e. Mereka melakukan percobaan sampai 3 kali.

21. Penggunaan tanda baca yang tidak sesuai dengan EyD adalah…. a. Saya tidak akan membeli mobil itu, karena harganya mahal. b. Oleh karena itu kalian harus rajin menabung.

Page 3: SBMPTN REVISI

c. Atas bantuan Saudara saya mengucapkan terima kasih. d. Hari sudah sore, tetapi pekerjaannya belum selesai juga. e. Semua siswa baik yang pria maupun wanita, mengikuti ujian bersama.

22. Kesalahan penerapan EyD terdapat pada pilihan jawaban…. a. Banjir yangmelanda kota Jakartadan sekitarnya hingga kemarin belum surut, sehingga semakin menambah

peneritaan ratusan ribu warga. b. Di beberapa tempat, masih banyak warga belum berhasil dievakuasi dari tempat tinggalnya. c. Banjir juga melanda Kabupaten Tegal. Akibatnya, sebuah gedung sekolah dan halamannya terendam lumpur. d. Seorang koreografer tidak boleh main-main dan asal-asalan dalam menggarap jika diserahi sebuah event. e. Seperti layaknya keraton-keraton di Pulau Jawa, Keraton Sumenep juga memiliki sebuah masjid Agung yang

dikenal sebagai masjid Jamik. 23. Pemakaian tanda baca koma (,) yang sesuai denga EyD adalah….

a. Tahun lalu, ia merayakan pesta pernikahan di hotel “berbintang lima” yang menghabiskan dana sekitar sepuluh miliar rupiah.

b. Karena sakit, setiap karyawan perusahaan , baik karyawan lama maupun karyawan baru diperkenankan tidak masuk kerja.

c. Karena reformasi, perusahaan yang akan beroperasi-mudah-mudahan dapat berjalan lancar-tidak dipersyaratkan mengurus surat izin yang bermacam-macam.

d. Lembaga pendidikan, khususnya lembaga pendidikan dasar seharusnya tidak “hanya” mementingkan kuantitas, tetapi juga kualitas.

e. Lembaga pendidikan dasar, menengah, dan atas perlu meningkatkan mutu proses pembelajaran, baik di dalam kelas maupun di luar kelas.

II. TATA KALIMAT (SINTAKSIS) 1. Peningkatan mutu pendidikan di Indonesia memerlukan beberapa komponen, antara lain, misalnya, guru yang

berkualitas, sarana dan prasarana yang lengkap, metode pembelajaran yang bervariasi, dan lain-lain. Kalimat tersebut bukan kalimat efektif. Perbaikannya adalah …. a. Peningkatan mutu pendidikan di Indonesia memerlukan beberapa komponen, antara lain guru yang berkualitas,

sarana dan prasarana yang lengkap, metode pembelajaran yang bervariasi. b. Peningkatan mutu pendidikan di Indonesia memerlukan beberapa komponen, antara lain, guru yang

berkualitas, sarana dan prasarana yang lengkap, metode pembelajaran yang bervariasi, dan lain-lain. c. Peningkatan mutu pendidikan di Indonesia memerlukan beberapa komponen, seperti, misalnya, guru yang

berkualitas, sarana dan prasarana yang lengkap, metode pembelajaran yang bervariasi, dan lain-lain. d. Peningkatan mutu pendidikan di Indonesia memerlukan beberapa komponen,yaitu antara lain: guru yang

berkualitas, sarana dan prasarana yang lengkap, metode pembelajaran yang bervariasi, dan lain-lain. e. Peningkatan mutu pendidikan di Indonesia memerlukan beberapa komponen: guru yang berkualitas, sarana

dan prasarana yang lengkap, metode pembelajaran yang bervariasi, dan lain-lain. 2. Kalimat berikut tidak memiliki gagasan yang jelas, kecuali….

a. Dalam Undang-undang No. 23 Tahun 2002, mengatur tentang berbagai kondisi anak- anak yang berkaitan dengan kondisi atau keberadaan orang tua.

b. Meski bentuk negara federal masih boleh dianggap sebatas wacana, bahkan memicu semangat beberapa daerah, terutama yang memiliki sumber kekayaan alam untuk menyetujuinya.

c. Sejak krisis ekonomi juga ditandai dengan munculnya berbagai konflik di beberapa daerah di indonesia. d. Kini telah dikembangkan teori ilmu pengetahuan sosial tentang media. e. Yang tidak dapat dipungkiri pula bahwa diadopsinya gagasan untuk pembentukan MK atas pengaruh

perkembangan hukum tata negara di Eropa.3. 1) Tembakau berkualitas baik versi petani boleh jadi dikategorikan sedang dan rendah oleh pengusaha. 2) …, petani

umumnya bertekuk lutut oleh vonis harga jual yang ditentukan pengusaha. Kata penghubung yang tepat untuk menggabungkan kedua kalimat tersebut adalah …. a. Sebab itu b. Sehingga c. Oleh karena itu d. Sebab e. Jadi

4. Yang merupakan kalimat efektif adalah… a. Dalam bab ini, akan menelusuri kasus malpraktik. b. Berita musibah gempa itu saya sudah sampaikan pada Pak Lurah. c. Bagi segenap pelajar yang akan mengajukan proposal penelitian harap mengirimkan datanya segera. d. Beberapa artikel-artikel ilmiah itu dimuat dalam jurnal ilmiah. e. Makalah ini membahas masalah kesehatan masyarakat di daerah rawan gempa.

5. Saat berlaga di Yuan Shen of Pudong, Shanghai, Taufik menyerah dua set langsung dari pemain tuan rumah. Inti kalimat tersebut adalah…. a. Taufik berlaga di Yuan Shen of Pudong, Shanghai. b. Kekalahan Taufik dari pemain tuan rumah. c. Pemain tuan rumah menang atas Taufik. d. Pertandingan antara Taufik dan pemain tuan rumah. e. Taufik menyerah dua set langsung.

6. (a) Teater tradisional berlatar belakang kebudayaan timur. (b) Teater modern berlatar belakang kebudayaan barat. Kedua kalimat tersebut dapat digabungkan menjadi….

Page 4: SBMPTN REVISI

a. Teater tradisisonal belatar belakang kebudayaan timur sehingga teater modern berlatar belakang kebuadayaan barat.

b. Teater tradisisonal belatar belakang kebudayaan timur, sedangkan teater modern berlatar belakang kebuadayaan barat.

c. Teater tradisisonal belatar belakang kebudayaan timur jadi teater modern berlatar belakang kebuadayaan barat.

d. Teater tradisisonal belatar belakang kebudayaan timur bahkan teater modern berlatar belakang kebuadayaan barat.

e. Teater tradisisonal belatar belakang kebudayaan timur apalagi teater modern berlatar belakang kebuadayaan barat.

7. Meskipun Indonesia merupakan negara pengekspor minyak bumi, tetapi pemerintah merasa berkewajiban untuk melkasanakan program hemat energi sejak tahun 2000. Agar menjadi kalimat efektif, kalimat tersebut dapat diperbaiki dengan cara…. a. Menghilangkan kata tetapi b. Memberikan tanda koma (,) setelah kata meskipun c. Menghilangkan tanda baca koma (,) setelah kata bumi d. Menghilangkan kata untuk e. Mengganti kata sejak dengan dari

8. Untuk memberikan pelayanan bagi para wisatawan yang umumnya adalah keluarga, penduduk setempat membangun dan menyewakan puluhan penginapan sederhana yang terbuat dari kayu beratap rumbia. Inti kalimat tersebut adalah…. a. Untuk memberikan pelayanan bagi para wisatawan yang umumnya adalah keluarga. b. Penduduk setempat membangun dan menyewakan puluhan penginapan sederhana yang terbuat dari kayu dan

beratap rumbai. c. Untuk memberikan pelayanan bagi para wisatawan yang umumnya adalah keluarga, penduduk setempat

membangun dan menyewakan puluhan penginapan sederhana.d. Penduduk setempat membangun dan menyewakan puluhan penginapan sederhana. e. Para wisatawan menyewa penginapan sederhana.

9. Lingkungan fisik Indonesia yang terdiri atas daerah-daerah yang terentang dari Sabang sampai Merauke yang bertabur beribu-ribu pulau di seluruh Nusantara berpengaruh pada praktik sosial, antara lain berupa perilaku bermasyarakat. Kalimat luas tersebut dibangun oleh beberapa kalimat tunggal berikut ini, kecuali…. a. Lingkungan fisik Indonesia terdiri dari daerah-daerah. b. Lingkungan fisik Indonesia terentang dari Sabang sampai Merauke. c. Daerah-daerah tertabur menjadi beribu-ribu pulau. d. Lingkungan fisik Indonesia berpengaruh pada praktik sosial. e. Praktik sosial berupa perilaku bermasyarakat.

10. Kurangnya tenaga kepariwisataan yang profesional itu, baik untuk level manajerial maupun bukan, tampaknya berkaitan dengan prospek karier yang mungkin belum banyak diperhitungkan selama ini. Kalimat inti kutipan tersebut adalah… a. Tenaga kepariwisataan yang profesional berbagai level itu kurang. b. Kurangnya tenaga kepariwisataan itu berkaitan dengan prospek karier. c. Masalah itu berkaitan dengan prospek karier. d. Masalah prospek karier itu belum banyak diperhitungkan. e. Tenaga kepariwisataan itu belum diperhitungkan.

11. Makalah ini akan berusaha menggali berbagai kekuatan ekspresi seni yang pada gilirannya”disepakati” sebagai sebuah produk yang tidak hanya mempresentasikan identitas individu, tetapi lebih banyak mempresentasikan identitas kelompok. Kalimat inti dari kalimat luas tersebut adalah…. a. Makalah ini menggali ekspresi seni.b. Ekspresi seni sebagai produk. c. Makalah berusaha menggali. d. Kekuatan ekspresi seni disepakati. e. Ekspresi seni mempresentasikan identitas.

12. Pada tahun ini banyak pekerja yang diberhentikan karena kasus meoneter yang tidak menentu. Inti kalimat tersebit adalah… a. Tahun ini banyak pekerja yang deberhentikan. b. Banyak pekerja diberhentikan. c. Pekerja diberhentikan. d. Karena kasus moneter tidak menentu. e. Kasus moneter tidak menentu.

13. Rangkaikan penggalan kalimat nomor 1,2,3,4,5, dan 6 ini sehingga menjadi kalimat yang baku dan lazim!1. menikmati keindahan2. dengan kehadiran Transit Planet Venus 3. masyarakat yang gemar 4. tanggal 8 Juni 2004

Page 5: SBMPTN REVISI

5. fenomena alam di langit 6. akan dimanjakan lagi Urutan yang tepat adalah…. a. 3,5,6,1,2,4 b. 3,2,4,6,1,5 c. 4,3,5,6,1,2 d. 3,1,5,6,2,4 e. 3,1,6,5,2,4

14. Kalimat yang paling tepat susunannya adalah… a. Jika Anda memerlukan bantuan dapat menghubungi kami di kantor. b. Bagi yang berminat mengikuti pelatihan musik dapat mendaftarkan diri di sekretariat. c. Dari hasil rapat kemarin mengisyaratkan bahwa proposal yang diajukan disetujui. d. Jika hal itu diperhatikan oleh pengguna jalan, tidak akan menimbulkan kemacetan. e. Dalam buku petunujk teknis, telah dijelaskan langkah-langkah kerja secara operasional.

15. Sikap negara-negara barat yang melarang warganya bepergian ke ASEAN, terutama ke Indonesia, dikritik oleh para pemimpin ASEAN dengan menyebutkan itu sebagai hal yang tidak perlu. Inti kalimat tersebut adalah…. a. Negara barat melarang warganya pergi ke ASEAN. b. Negara barat tidak perlu melarang warganya pergi ke ASEAN. c. Sikap negara barat dikritik pemimpin ASEAN. d. Sikap negara barat merupakan hal yang tidak perlu. e. Negara barat melarang warganya.

16. 1) Sebagian nasabah masih saja resah. 2) Pemerintah memberikan jaminan bahwa uang nasabah tetap aman. Kata penghubung yang tepat untuk menggabungkan kedua kalimat tersebut adalah… a. jika b. agar c. walaupun d. sementara itu e. sebaliknya

17. Susunlah ketujuh bagian kalimat beriktu menjadi sebuah kalimat yang baik! 1) yang tinggal di pedalaman 2) bahaya bekerja di tengah laut 3) pekerjaan tersebut 4) sebagai pengawas jaring di jermal 5) dan tidak mengetahui 6) cukup menarik perhatian 7) bagi anak desa Susunan yang benar adalah….a. 4,3,6,7,1,5,2 b. 4,1,5,2,3,6,7 c. 7,1,3,4,6,5,2 d. 7,1,5,2,4,3,6 e. 6,3,7,4,1,5,2

18. Kalimat-kalimat berikut memiliki pola yang sama, kecuali…. a. Anton menarik lengan saya seraya menunjuk ke sebuah mobil BMW yang sedang diperbaiki. b. Peristiwa itu terjadi sewaktu keluargaku sedang dalam suasana berkabung. c. Aku lebih bergembira sejak sikap ibu padaku berubah. d. Ia baru kembali ke desa setelah biaya untuk melanjutkan sekolahnya tidak ada. e. Aku melompat dari anak tangga dan kemudian berlari ke halaman.

19. Kakak mempelajari bahasa Jerman selama tiga tahun. Pola kalimat tersebut sepola dengan kalimat…. a. Paman berdagang buah-buahan di Pasar Wage. b. Mereka berdoa dengan khusuk kepada Tuhannya. c. Orang itu minum bir tadi malam. d. Adik belajar silat di Perguruan Elang putih. e. Anak kecil itu menangis di pangkuan ibunya.

20. Masalah banjir adalah masalah nasional. Kalimat tersebut berpola…. a. S-P-O b. S-P c. S-P-K d. P-O-S e. P-S-O

21. Subjek dari kalimat Dulu, stadion itu tempat balapan anjing , adalah….a. dulu b. stadion itu c. tempat d. balapan e. anjing

22. Masyarakat tidak perlu khawatir akan persediaan sembilan bahan pokok . Kalimat di bawah ini yang sepola dengan kalimat di atas adalah …. a. Pemerintah tidak akan mengadakan penjadwalan kembali utang luar negeri. b. Kondisi itu tidak akan menambah perkembangan industri ternak. c. Hampir semua negara tidak peduli akan pajak hasil ternak. d. Usaha ternak akan terhambat kalau dikenakan PPN sepuluh persen. e. Peranan industri makanan ternak cukup strategis.

23. Pendidikan budu pekerti lebih difokuskan pada pembentukan pribadi luhur, …pendidikan kewarganegara- an difokuskan pada pembentukan watak kebangsaan. Kata penghubung yang tepat untuk melengkapi kalimat tersebut adalah… a. sedangkan b. padahal c. namun d. akan tetapi e. sebaliknya

24. Para ahli memperkirakan bahwa hari ini berawan. Kalimat berikut yang sepola dengan kalimat di atas adalah… a. Darmawan menyuruh saya beristirahat. b. Kami telah memperbaiki sepeda yang rusak. c. Sartono menghadiahi Rinawati kado ulang tahun.

Page 6: SBMPTN REVISI

d. Hal itu akan menjadi tanggung jawab kami. e. Kegiatan ini merupakan program akhir tahun.

25. Adik kecil menangis di pangkuan ibu. Pola kalimat berikut yang sama dengan pola kalimat di atas adalah… a. Adik kecil disusui oleh ibu. b. Bayi ini lahir di rumah sakit bersalin. c. Tetangga saya melahirkan bayi kembar. d. Bibi Aminah mengangkat anaknya dengan hati-hati. e. Di desa sudah banyak bidan membantu ibu-ibu melahrikan.

III. PARAGRAF Teks berikut untuk menjawab soal nomor 1 s.d. 5. Bacalah dengan cermat!

Pemerintah Kabupaten Malang saat ini tengah mencari investor untuk membangun pabrik semen kawasan Malang Selatan. Diperkirakan, investasi pabrik mencapai Rp 1,4 triliun. Menurut Kepala Dinas Lingkungan Hidup, Energi, dan Mineral Kabupaten Malang, Budi Iswoyo, Pemkab Malang tidak sanggup mendanainya.” Oleh karena itu, kami sangat membutuhkan suntikan dana dari pihak ketiga yang berminat menanamkan investasinya di sini. “ ujarnya, Jumat (13/1) Saatini, Budi menerangkan pihaknya sudah mempresentasikan rencana mendirikan pabrik tersebut ke beberapa pengusaha semen di Sulawesi melalui PT Semen Gresik. Namun, sejauh ini belum ada yang dapat memberikan kepastian. Berdasarkan studi kelayakan yang telah dilakukan, potensi batu kapur yang merupakan bahan utama semen ini terdapat di lima kecamatan Sumbermanjing Wetan, Gedangan, Dampit, dan Pagak. Sekretaris Asosiasi Perusahaan Tambang (Apertam) Jawa Timur, M. H. Rudin Al- Sonny Y.P.E., mengatakan bahwa sebelum memilkirkan pendirian pabrik semen, Pemkab Malang seharusnya mendahulukan pembangunan pelabuhan di Malang Selatan. Pelabuhan ini sangat penting peranannya, baik dalam hal bongkar muat batu bara sebagai bahan pabrik, maupun transportasi hasil produksi pabrik.

1. Pokok pikiran yang terdapat pada teks tersebut adalah . . . .a. Kebutuhan dana investasi Pemkab Malangb. Daerah penghasil kapur di Kabupaten Malangc. Rencana Pemkab Malang membangun pabrik semen di kawasan Malang Selatan.d. Tata ruang sentra produksi di Kabupaten Malang.e. Kebutuhan investasi mencapai Rp 1,4 triliun.

2. Sebelum merencanakan pembangunan pabrik semen, pembangunan infrastruktur yang seharusnya didahulukan adalah . . . .a. Pembangunan jalan lintas Malang Selatan.b. Pembangunan pelabuhan di Malang Selatan.c. Studi kelayakan pemetaan wialayah kecamatan penghasil kapur.d. Pembuatan pabrik batu bara bahan bakar pabrik.e. Pembelian material bangunan fisik pabrik.

3. Pada teks di atas, yang dimaksud investor adalah . . . .a. Pemerintah Kabupaten Malangb. PT Semen Gresikc. Asosiasi Perusahaan Tambang (Apertam)d. Pihak ketiga yang berminat menanamkan investasie. Warga Malang Selatan

4. Gagasan utama paragraf kedua pada teks di atas adalah . . . .a. Rencana pembangunan pabrik semen di Malang Selatanb. Pemkab Malang tidak mampu mendanai pembangunan pabrik semen di Malang Selatanc. Prioritas pembangunan di Malngd. Malang Selatan sebagai penghasil batu kapur sebagai bahan bakar semene. Penawaran rencana pembangunan pabrik semen ke PT gresik

5. Arti kata “produksi” pada teks di atas adalah . . . .a. Proses mengolah batu kapur menjadi semenb. Pembangunan pabrik semen di Malang Selatanc. Kegiatan penawaran rencana Pemkab Malang utnuk membangun pabrik semen kepada pihak ketigad. Pengangkutan batu bara sebagai bahan bakar pabrike. Ketidakmampuan nPemkab malang mendanai pembangunan pabrik semen di Malang Selatan

Teks berikut untuk soal nomor 6 s.d 9. Bacalah dengan cermat!1) Semua orang pasti mengenal pendidikan. 2) Pendidikan dalah proses internalisasi budaya ke dalam diri

seseorang dan masyarakat sehingga membuat orang dan masyarakat jadi beradab. 3) Pendidikan bukan hanya merupakan sarana transfer ilmu pengetahuan, tetapi lebih luas lagi, yakni sebagai sarana pembudayaan dan npenyaluran nilai (enkulturisasi dan sosialisasi). 4) Anak harus mendapatkan pendidikan yang menyentuh dimensi dasar kemanusiaan. 5) Dimensi kemanusiaan itu mencakup sekurang- kurangnya tiga hal penting mendasar.

6) Pendidkan karakter adalah pendidikan budi pekerti plus, yaitu yang melibatkan aspek pengetahuan, perasaan, dan tindakan. 7) Menurut Lickona, tanpa ketiga aspek itu, pendidikan karakter tidak akan efektif. 8) Dengan pendidikan karakter yang diterapkan secara sistematis dan berkelanjutan, seorang anak akan menjadi

Page 7: SBMPTN REVISI

cerdas emosinya. 9) Kecerdasan emosi ini adalah bekal penting dalam mempersiapkan anak menyongsong masa depan. 10) Terdapat sembilan pilar karakater yang berasal dari nilai- nilai luhur universal, yaitu karakter cinta Tuhan dan segenap ciptaan- Nya; kemandirian dan tanggung jawab; kejujuran/ amanah dan diplomatis; hormat dan santun; dermawan, suka menolong, dan gotong royong/ kerja sama, percaya diri dan pekerja keras; kepemimpinan dan keadilan; baik dan rendah hati; serta toleran dan cinta damai. (Dikutip dengan pengubahan dari blogdetik.com)

6. Gagasan utama paragraf I pada teks tersebut adalah . . . .a. Pendidikan dikenal setiap orangb. Pendidikan adalah internalisasi budayac. Pendidikan bukan sarana transfer ilmud. Pendidikan merupakan sarana pembudayaane. Pendidikan harus berdimensi kemanusiaan

7. Pertanyaan berikut yang jawabannya terdapat pada paragraf kedua adalah . . . .a. Mengapa pendidikan karakter merupakan pendidikan budi pekerti?b. Mengapa kecerdasan emosi penting dalam mempersiapkan masa depan?c. Bagiamana pendidikan karakter yang sistematis dan berkelanjutan?d. Bagaimana pendapat Lickona tentang aspek pendidikan karakter?e. Siapa yang mengemukakan sembilan pilar karakter berasal dari nilai luhur universal?

8. Apa perbedaan gagasan antara paragraf kesatu dan paragraf kedua?a. Paragraf pertama memaparkan pendidikan karakter secara umum, sedangkan paragraf kedua memaparkan

pendidikan karakter secara khusus.b. Paragraf pertama memaparkan konsep pendidikan karakter , sedangkan paragraf kedua memaparkan unsur

pendidikan karakte.c. Paragraf pertama memaparkan pendidikan karakter secara umum, sedangkan paragraf kedua memaparkan

konsep pendidikan karakter.d. Paragraf pertama memaparkan konsep pendidikan karakter secara umum, sedangkan paragraf kedua

memaparkan pendidikan karakter secara khusus.e. Paragraf pertama memaparkan pendidikan secara umum, sedangkan paragraf kedua memaparkan manfaat

pendidikan karakter.9. Apa kelemahan isi paragraf pertama?

a. Tidak ada hubungan antara penjelasan kecerdasan emosi (kalimat9) dengan sembilan karakter (kalimat 10)b. Plus pada pendidikan budi pekerti (kalimat 6) tidak dijelaskan secara rinci pada bagian berikutnya.c. Penjelasan tentang pendidikan sebagai sarana transfer ilmu ( kalimat 3) bertentangan dengan konsep

pendidikan (kalimat 2)d. Tidak hubungan antara ketiga aspek yang dimaksud Lickona (kalimat 7) dengan nilai plus pada pendidikan budi

pekerti (kalimat 6).e. Penjelasan tentang kecerdasan emosi ( kalimat 8) tidak berkaitan dengan pendidikan karakter ( kalimat 7).

Teks berikut untuk soal nomor 10 s.d 14. Bacalah dengan cermat!1) Motivasi berasal dari kata motove atau movere yang berarti “mengerahkan”. 2) Seperti yang dikatakan Liang Gie

(dalam Martoyo, 2000), motif adatau dorongan adalah suatu dorongan yang menjadi pangkal seseorang melakukan sesuatu atau bekerja. 3) Orang yang sangat termotivasi adalah orang yang melaksanakan upaya substansial guna menunjang tujuan –tujuan produksi kesatuan kerjanya dan organisasi diman dia bekerja. 4) Seseorang yang tidak termotivasi hanya memberikan upaya minim ketika bekerja.

5) konsep motivasi merupakan sebuah konsep penting dalam studi tentang kinerja individual. 6) Dengan demikian, motivasi berarti pemberian motif atau hal yang menimbulkan dorongan. 7) Dapat juga dikatakan bahwa motivasi adalah faktor yang mendorong orang utnuk bertindak dengan cara tertentu.

8) Menurut Martoyo (2000) manusia akan memiliki semangat untuk mengerjakan suatu hal jika dapat menghasilkan sesuatu yang dianggapnya sangat berharga yang berdampak pada kelangsungan hidupnya, menimbulkan rasa tenteram, rasa aman, dan sebagainya. 9) Motivasi atau dorongan kepada pegawai untuk bersedia bekerja sama demi tercapainya tujuan bersama dapat diklasifikasi menjadi dua maacam, yaitu motivasi finansial dan nonfinansial

10) Motivasi finansial adalah dorongan yang dilakukan dengan memberikan imbalan finansial kepada pegawai. 11) Imbalan tersebut sering disebut dengan istilah insentif. 12) Motivasi nonfinansial adalah dorongan yang tidak diwujudkan dalam bentuk finansial atau uang, melainkan diwujudkan dalam bentuk pujian, pengahrgaan, pendekatan antara manusia, dan lain sebagainya. 13) Dari penjelasan ini dapat dismpulkan bahwa motivasi sangat penting bagi suatu perusahaan atau instansi karena dapat mendorong karyawan untuk bertindak secara ikhlas dalam mencapai tujuan perusahaan secara efisien.

(Dikutip dengan perubahan dari ridwanrubic.wordpress.com)10. Penggunaan kata yang tidak tepat terdapat pada kata . . . .

a. Minim dalam kalimat 4b. Motif dalam kalimat 6c. Substansial dalam kalimat 3d. Individual dalam kalimat 5e. Efisien dalam kalimat 3

11. Kata ini pasa kalimat 13 merujuk pada . . .a. Motivasi finansial dan nonfinansialb. Dampak motivasi d. pemberian motifc. Imabalan finansial e. konsep dan jenis motivasi

Page 8: SBMPTN REVISI

12. Kesalahan penggunaan ejaan ditemukan pada kalimat ….a. 1 dan 6 d. 5 dan 7b. 2 dan 12 e. 8 dan 10c. 3 dan 9

13. Bagaiman hubungan isi antarparagraf dalam teks tersebut?a. Paragraf kesatu menjadi penyebab paragraf keduab. Paragraf kedua menjadi penyebab paragraf ketigac. Paragraf kesatu menjadi pembanding paragraf kesatud. Paragraf ketiga merupakan contoh paragraf kesatue. Paragraf keempat menjelaskan paragraf ketiga

14. Mengapa di sebuah organisasi ada karyawan yang tidak maksimal dalam bekerja?a. Finansial yang diterima karyawan tidak sepadan dengan tugasnya.b. Tujuan organisasi belum menjadi tujuan bersama karyawan.c. Imbalan karyawan belum sepadan dengan beban kerjanya.d. Organisasi belum memperhatikan finansial dan nonfinansial.e. Motivasi yang dilakukan organisasi kepda karyawan tidak tepat.

Teks brikut untuk soal nomor 15 s.d. 20. Bacalah dengan cermat!1) Pembentukan masyarakat ekonomi ASEAN 2015 membuka peluang sekaligus tantangan bagi Indonesia dalam

meningkatkan kesejahteraan bangsa melalui integrasi ekonomi dan keuangan di kawasan. 2) Luasnya wilayah Indonesia dengan komoditas unggulan ekspor relatif bervariasi antarprovinsi membuka peluang peningkatan diversifikasi ekspor Indonesia, baik dalam rangka meningkatkan ekspor ke kawasan ASEAN, Asia lainnya maupun dunia. 3) Secara spesifik, ekspor Indonesia ditentukan oleh kinerja ekonomi dan perdagangan provinsi. 4) Perdagangan internasional di suatu negara dapat dibangun oleh perdagangan dari setiap provinsi.

5) Unggulan ekspor yang relatif, bervariasi antarprovinsi dapat didayagunakan untuk memacu pertumbuhan ekspornya. 6) Hingga saat ini, provinsi yang paling dominan malakukan kegiatan ekspor adalah Jawa Barat diikuti Riau, Jawa Timur, dan Jakarta. 7) Sebanyak 26,2 % ekspor Jabar ditujukan ke ASEAN dengan negara tujuan utama Malaysia, Filipina, Thailand, dan Vietnam. 8) Provinsi Riau melakukan kegiatan ekspor terutama ke kawasab Sinagpura.

9) Jika dilihat secara lebih detail, maka setiap wilayah di Indonesia memiliki kekhususan komoditas utama yang diekspor, misalnya Sumatera: mianyak sawit dan karet mentah; Kalimantan: batu bara; Sulawesi: cokelat, minyak sawit, dan metalliferous; serta Irian: metalliferous. 10) Dari data pertumbuhan investasi dan kinerja ekonomi makro Indonesia diketahui sebgian besar provinsi di Indonesia hanya memilikikomoditas ekspor utama terbatas pada satu hingga dua komoditas. 11) Hanya beberapa provinsi saja di Pulau Jawa yang memiliki komoditas ekspor unggulan yang lebih terdiversifikasi.

Tabel Kontribusi Provinsi terhadap EksporProvinsi Ekspor (%)

2005 2006 2007Jawa Barat 23,2 21,4 14,4Riau 13,6 13,0 13,5Jawa Timur 9,9 10,4 11,5Jakarta 8,1 7,9 7,8Sumut 6,6 6,4 6,5Kaltim 5,9 6,2 5,9Banten 7,7 7,6 6,6Jawa Tengah 4,2 3,9 3,8Irian 3,8 3,9 4,8Kalsel 3,1 3,7 3,8

(Dikutip dengan perubahan dari Outlook Ekonomi Indonesia 2008- 2011)15. Pernyataan yang paling sesuai dengan isi tabel tersebut adalah . . .

a. Hanya Provinsi Jawa Timur yang menunjukkan perkembangan positif ekspor nonmigas.b. Hanya Provinsi Jawa Barat yang menunjukkan persentase ekspor menurun dari tahun 2005 hingga tahun

2007.c. Provinsi yang paling dominan melakukan kegiatan ekspor adalah provins- provinsi di Pulau Jawa.d. Provinsi Riau, Sumut, dan Kaltim mengalami mengalami penurunan ekspor pada tahun2006 dan kembali

bangkit pada tahun 2007.e. Kontribusi ekspor nonmigas Provinsi Riau melebihi akumulasi provinsi- provinsi di Pulau Kalimantan.

16. Kalimat yang tidak efektif pada teks tersebut terdapat pada kalimat nomor . . . .a. 1 dan 3 d. 5 dan 10b. 2 dan 6 e. 7 dan 11c. 4 dan 9

17. Simpulan yang tepat untuk teks tersebut adalah . . . .a. Komoditas ekspor Indonesia yang sangat bervariasi memacu peningkatan diversifikasi ekspor ke dunia.b. Setiap wilayah di indonesia memiliki kekhususan komoditas utama yang diekspor ke kawasan ASEAN.c. Pembentukan masysarakat ekonomi ASEAN 2015 memacu pertumbuhan ekspor Indonesia.d. Sebagian besar provinsi di Indonesia hanya memiliki komoditas ekspor utama yang terbatas.

Page 9: SBMPTN REVISI

e. Peningkatan kesejahteraan bangsa dapat dialkukan melalui pengintegrasian ekonomi dan keuangan kawasan.

18. Penulisan teks tersebut bertujuan agar pembaca . . . .a. Meyakini bahwa komoditas ekspor nonmgas di Indonesia penting untuk diperhatikan lebih seksama ke

depannya.b. Memiliki gambaran bahwa potensi ekspor nonmigas Indonesia lebih besar daripada ekspor migas.c. Menyimpulkan bahwa beberapa provinsi di Jawa memiliki komoditas ekspor unggulan yang lebih

terdiversifikasi.d. Menyadari bahwa kevariasian unggulan produk provinsi harus didyagunakan untuk meningkatkan

pertumbuhan ekspor.e. Menyadari bahwa pada tahun 2015 anggota ASEAN siapmenghadapau kerja sama ekonomi antaranggota.

19. Apa kelamahan isi teks tersebut?a. Pengintegrasian ekonomi dan keuangan tidak diikuti penjelasan tentang wujud pengintegrasian tersebut.b. Pernyataan bahwa produk ekspor antarprovinsi bervariasi tidak didukung oleh komoditas ekspor tiap

provinsi.c. Dominasi ekspor Jawa Barat tidak disertai penjelasan tentang seberapa besar persentase ekspor tersebut.d. Isian tabel tidak menjelaskan perkembangan kontribusi ekspor provinsi dan perbandingannya antarprovinsi.e. Tantangan untuk meningkatkan ekspor Indonesia tidak diikuti penjelasan tentang faktor penentu ekspor

tersebut.20. Skema yang paling tepat untuk menggambarkan teks tersebut adalah . . . .

a.

b.

c.

d.

ke

Ekspor Indonesia

integrasi

Kinerja Ekonomi

ekonomi

Keuangan

Kinerja perdagangan

Indonesia

Provinsi pengeksporan

Jawa Kalimantan Irian

Ekspor Indonesia

ASEAN ASIA Internasio-nal

Pembentukan Masyarakat Ekonomi ASEAN

Ekonomi dan keuangan

Komoditas unggulan

Kesejahteraan bangsa

Page 10: SBMPTN REVISI

e.

21. Bacalah teks berikut dengan cermat!Berita yang dianalisir Antara bulan lalu dengan tajuk “ Bahasa Indonesia Berpeluang Jadi Bahasa Kedua

ASEAN” memberikan satu harapan besar bagi Indonesia untuk berbicara di kancah internasional. Sekarang ada kesadaran di kalangan warga Philipina – teristimewa di kawasan selatan negara ini- bahwa mereka perlu belajar bahasa Indonesia karena bahasa itu dapat dimengerti di empat negara ASEAN lainnya: Brunei, Malaysia, Singapura, dan Thailand (Selatan). Selain keempat negara ASEAN itu, sebagian warga suku Champ di Kamboja, Laos, dan Vietnam mengerti bahasa Melayu yang menjadu induk bahasa Indonesia . . .

Kalimat yang tepat untuk menutup paragraf tersebut adalah . . . .a. Untuk itu, perlu menggalakkan penyebaran kamus bahasa Indonesia agar memudahkan orang asing.b. Dengan demikian, jelaslah bahwa bahasa Indonesia menjadi bahasa yang diidamkan di kawasan ASEAN.c. Oleh karena itu, bangsa Indonesia harus dapat menggunakan bahasa Indonesia dengan baik dan benar.d. Untuk itu, perlu diupayakan agar bahasa Indonesia menjadi bahasa utama ASEAN.e. Sehubungan dengan hal itu, menjunjung tinggi bahasa Indonesia menjadi kewajiban seluruh bangsa Indonesia.

22. Seseorang akan mengembangkan karangan yang bertema upaya meningkatkan kemampuan berwira usaha bagi remaja dengan kerangka sebagai berikut:1) Saran bagi remaja untuk berwira usaha.2) Pentingnya meningkatkan kemampuan berwira usaha.3) Langkah meningkatkan kemampuan berwira usaha.4) Komponen yang mendukung kemampuan berwira usaha. 5) Ketidakmampuan remaja berwira usaha.

Urutan kerangka yang paling tepat untuk tema karangan di atas adalah . . . .a. 5,1,2,4, dan 3 d. 4,5,2,1, dan 3b. 2,4,5,1, dan 3 e. 5,2,4,3, dan 1c. 2,3,4,5, dan 1

23. Bacalah kedua teks berikut dengan cermat!Teks I

Obat telah menjadi bagian yang tidak terpisahkan dari ehidupan manusia masa kini. Bahkan, karena begitu akrabnya, orang justru semakin tergantung pada obat, semakin terbiasa menggunakan obat. Oenggunaan obat- obatan kini justru terlihat sebagai gaya hidup modern. Mereka perlu tampil seksi, bukan sehat, perlu obat kuat bukan hidup harmonis,dan lain- lain. Faktanya zat- zat yang terkandung dalam obat dan efeknya bagi tubuh kita sering tidak kita perhatikan karena menganggap obat itu menyembuhkan tanpa memperhatikan apa yang sebenarnya terkandung dalam obat tersebut. Dalamp[emilihan obat harus diperhatikan adanya kandungan bahan- bahan yang justru menimbulkan dampak buruk terhadap tubuh kita.

Teks IIPada umumnya, orang memandang obat sebagai sesuatu yang ajaib yang dapat menyembuhkan segala

penyakit dengan cepat tanpa efek samping. Semua obat adalah ra cun, kecuali diminum sesuai dengan dosis yang diizinkan. Hal itu menyadarkan kita bahwa dalam mengonsumsi obat kita harus memperhatikan dosis yang tepat untuk kesembuhan yang diaharapkan. Masyarakat menaruh harapan besar akan obat, namunhal tersebut tidak sebanding dengan pengetahuanmasyarakat akan obat. Informasi tentang pentingnya obat, penggunaan, dan bahayanya sangat penting bagi masyarakat.

Kedua teks tersebut menjelaskan . . . .a. Kebutuhan manusia akan obat d. harapan manusia akan obatb. Cara mengonsumsi obat e. efek samping mengonsumsi obatc. Pengetahuan masyarakat akan obat

Masyarakat ekonomi ASEAN

Pendayagunaan ekspor yang bervariasi

Integrasi ekonomi dan keuangan kawasan

Kesejahteraan bangsa Indonesia

Page 11: SBMPTN REVISI

Teks berikut untuk soal nomor 24- 28. Bacalah dengan cermat!1) Kenaikan harga minyak dunia telah membuat selisih harga bahan bakar minyak (BBM) subsidi dan

nonsubsidi semakin melebar. 2) Keadaan ini, yang memicu peningkatan peralihan konsumsi dari BBM nonsubsidi ke BBM bersubsidi. 3) Tidak mengherankan jika realisasi subsidi BBM haingga akhir tahun ini diperkirakan menembus angka 41 juta kiloliter dari rencana yang dipatok APBN-P 2011 yang hanya 40,4 juta kiloliter.

4) Kondisi tersebut tentu tidak menguntungkan karena APBN jelas- jelas terbebani. 5) Subsidi BBM juga menciptakan ketidakadilan karena penikmat subsidi BBM adalah kelompok masyarakat menengah ke atas. 6) Di sisi lain, alokasi subsidi BBM yang besar menyebabkan anggaran belanja untuk sektor lainnya (termasuk bagi kelompok miskin) menjadi berkurang. 7) Apalagi dengan adanya pengendalian konsumsi BBM bersubsidi juga menimbulkan efektif negatif lainnya yang tidak kalah berat. 8) BBM bersusidi menimbulkan pemborosan dalam hal konsomsi energi tak terbarukan. 9) Kondisi ini sama saja dengan mempercepat proses terjadinya kelangkaan energi.

10) BBM bersubsidi hanya menguntungkan pemilik kendaraan pribadi sehingga menimbulkan persoalan pelik bagi sistem transportasi darat di kota- kota besar. 11) Akibatnya, industri transportasi massal menjadi tidak menarik, yang sebenarnya diaharapkan dapat memecahkan masalah keruwetan transportasi darat. 12) Di sisi lain, BBM bersubsidi menimbulkan ketidakadilan antarwilayah, karena penikmat subsidi BBM justru adanya di kota- kota besar dan golongan kaya.

13) Jelaslah, mempertahankan kebijakan subsidi BBM sebenarnya lebih banyak efek negatifnya dibandingkan dengan manfaatnya. 14) Oelh karena itulah, kebijakan pembatasan penggunaan BBM bersubsidi semestinya segera direalisasikan. 15) apabila tidak dibatasi, dikhawatirkan konsumsi BBM yang tentunya berpotensi menimbulkan amplikasi negatif yang lebih berat. 16) Kini di tangan pemerintahlah bola untuk menerapkan opsi pengaturan penggunaan BBM bersubsidi berada.

24. Kalimat yang tidak efektif ditemukan pada kalimat nomor . . . .a. 1 dan 5 d. 8 dan 15b. 3 dan 7 e. 10 dan 16c. 4 dan 14

25. Kesalahan penggunaan ejaan ditemukan pada kalimat nomor . . . .a. 1 dan 6 d. 5 dan 10b. 2 dan 12 e. 7 dan 16c. 4 dan 11

26. Rangkuman yang tepat untuk teks di atas adalah . . . .a. Kenaikan harga BBM dunia membebani APBN karena kebijakan subsidi BBM lebih banyak negatifnya sehingga

pembatasan penggunaan BBM bersubsidi semestinya segera direalisasikan.b. Kebijakan pengurangan subsidi BBM dalam APBN merupakan langkah yang bijak tidak hanya demi keadilan,

tetapi juga untuk menjaga kesehatan energi tak terbarukan pada masamendatang.c. Kebijakan pembatasan BBM bersubsidi merupakan langkah bijak demi kedailan guna mewujudkan kebijakan

membagi bebas subsidi BBM dari kelas menengah – atas ke kelas bawah.d. Kenaikan harga minyak dunia membebani APBN sehingga diperlukan langkah bijak demi kedailan karena yang

menimati subsidi BBM justru ada di kota- kota besar dan golongan kaya.e. APBN akan jelas- jelas terbebani karena konsumsi BBM bersubsidi pada tahun mendatang semakin besar

sehingga berpotensi menimbulkan implikasi negatif yang lebih berat.27. Hubungan isi teks antarparagraf dalam teks di atas yang paling gtepat adalah . . . .

a. Paragraf kedua menjadi penyebab paragraf kesatub. Paragraf ketiga merupakan contoh paragraf keduac. Paragraf ketiga menjadi pembanding paragraf keduad. Paragraf keempat merupakan rincian paragrafketigae. Paragraf kedua menjadi akibat paragraf kesatu

28. Penulisan teks tersebut berpihak kepada . . . .a. Pemilik kendaraan pribadi d. pengguna transportasib. Pengusaha transportasi e. masyarakatc. Pemerintah

Teks berikut untuk soal nomor 29 s.d. 32. Bacalah dengan cermat!Ilmuwan Universitas California Los Angeles, Amerika Serikat, memiliki hasil penelitian yang unik tentang bangsa

primata, yaitu primata memiliki wajah yang beragam. Dalam tiap wajah, tersimpan rahasia adaptasi dan evolusi setiap jenis primata. Ketika spesiae hidup dalm grup besar, wajahnya menjadi lebih sederhana dan polos. Hal ini berkaitan dengan kemampuan komunikasi dengan ekspresi wajah. Wajah yang lebih polos memungkinkan primata menunjukkan ekspresi yang lebih mudah.

Manusia dan orang utan merupakan dua jenis primata yang memiliki wajah polos. Pada manusia misalnya, tidak banyak oranamen dan warn, dan a yang dimiliki pda wajah, hanya kumis, rambut, dagu, alisBulu mata. Sementara itu, kenyataan tersebut terdengar biasa, ilmuwan justrunterkejut sebab mengira sebaliknya. Semula mereka mengira dalam grup besar, wajah akan sangat bervariasi dan kompleks sehingga memnungkinkan individu untuk membuat identifikasi anggota grupnya. Akan tetapi, bukan itu yang ditemukan. Spesies grup besar cenderung hidup berdekatan dan lebih memaki ekspresi wajah daripada grup kecil yang tersebar.

Sementara itu, wajah kompleks justru ditemukan pada jenis primata yang hidup dalam grup kecil. Kehidupan mereka cenderung soliter. Wajah yang kompleks memungkinkan setiap monyet dapat membedakan individu satu dengan lainnya. Wajah yang kompleks tersebut terlihat pada pola warna dan ornamen yang dimiliknya.

Page 12: SBMPTN REVISI

Jenis monyet laba- laba hitam yang memiliki wajah berpola warna hitam. Area sekitar mata berevolusi menjadi lebih hitam pada spesies yang hidup di tempat yang tinggi radiasinya. Marmoset emas memiliki wajah yang dipenuhi rambut- rambut panjang berwarna pirang. Hal itu mungkin berkaitan dengan habitatnya yang berada di daerah dingin. Rambut panjang membantu marmoset emas mempertahankan suhu tubuh tetap hangat . variasi wajah terebut menunjukkan pola adaptasi dan evolusi primata tersebut.29. Suntingan manakah yang paling tepat untuk kalimat ke-4 paragraf ke-2 di atas?

a. Semula mereka mengira dalam grup besar, wajah akan sangat bervariasi dan kompleks sehingga memungkinkan individu untuk melakukan identifikasi anggota grupnya.

b. Semula mereka mengira dalam grup besar, wajah akan sangat bervariasi dan kompleks, sehinggamemungkinkan individu untuk melakukan indentifikasi anggota grupnya.

c. Semula mereka mengira dalam grup besar, wajah akan sangat bervariasi dan kompleks sehinggamemungkinkan individu untuk melakukan indentifikasi anggota grupnya.

d. Semula , mereka meangira dalam grup besar, wajah akan sangat bervariasi dan kompleks, sehinggamemungkinkan individu untuk melakukan indentifikasi anggota grupnya.

e. Semula mereka meangira dalam grup besar, wajah akan sangat bervariasi dan kompleks, sehinggamemungkinkan individu melakukan indentifikasi anggota grupnya.

30. Gagasan utama paragraf keempat adalah….a. Jenis monyet laba- laba hitam memiliki wajah berpola warna hitam yang dihasilkan dari proses evolusi.b. Sekitar mata monyet laba- laba hitam menjadi lebih hitam karena berada di tempat yang beradiasi tinggi.c. Wajah marmoset emas yang hidup di daerah dingin akan memiliki rambut panjang dan pirang.d. Monyet laba- laba hidup di tempat beradiasi tinggi dan marmoset hidup di daerah dingin.e. Variasi wajah monyet menunjukkan pola adaptasi dan evolusi mereka terhadap lingkungannya.

31. Simpulan isi teks tersebut adalah . . . a. Kompleksitas perilaku primata ditunjukkan melalui berbagai bentuk ekspresi wajahnya.b. Primata memiliki wajah beragam yang diduga sebagai hasil adaptasi dan evolusi.c. Keberagaman wajah bangsa primata adalah hasil adaptasi, evolusi, dan perilaku mereka.d. Bentuk ekspresi wajah primata menjadi penanda perilaku sosial dan habitat mereka.e. Bentuk ekspresi wajah bangsa primata berhubungan dengan perilaku sosialnya.

32. Kelemahan paragraf pertama teks di atas adalah . . . .a. Gagasan utamanya terletak di tengahb. Kalimat penjelas tidak relevanc. Paragraf meangandung kalimat sumbangd. Jumlah kalimat penjelas kurange. Gagasan utamanya lebih dari satuTeks berikut untuk soal nomor 33 s.d 35. Bacalah dengan cermat!

Pengangguran adalah status yang paling tidak menyenangkan. Dia akan tertekan dari segala sisi, apalagi jika orang itu menyandang status pengangguran terdidik dengan embel- embel gelar sarjana. Sementara itu, penyelesainnya amat sederhana, yaitu Anda tinggal membuka usaha menjadi wirausahawan atau bekerja pada pengusaha lain. Namun, kenyataannya baru 0,18% penduduk Indonesia yang menjadi pengusaha. Artinya, baru sekitar 450.000 penduduk Indonesia yang membuka usaha dan tidak bekerja pada orang lain ( data majalah “Kesra” menyebutkan sebanyak 400.000 orang).

Andaikan setiap pengusaha mempu merekrut rata- rata 50 0rang pekerja, itu pun baru 22,5 juta orang yang dapat dipekerjakan. Sementara itu, angkatan kerja di Indonesia tidak kurang dari 100 juta orang, sedang angka pengangguran saat ini mencapai 10- 30 juta orang. Angka itu menunjukkan bahwa jumlah pengangguran masih banyak yang belum terserap di lapangan kerja. Data BPS menunjukkan bahwa jumlah pengangguran di Indonesia paling banyak ditempati oleh lulusan SLTA sebagaimana disajikan pada tabel berikut.

Tabel Data jumlah Penganguuran 2008-2011Tingkat Peendidikan 2008 2009 2010 2011

SLTP 1.973.986 1.770.823 1.657.452 1.661.449SLTA 3.812.522 3.879.471 3.446.137 3.344.325

Diploma 362.683 441.100 538.186 443.222Universitas 598.318 701.651 820.020 710.128

33. Pernyataan yang paling sesuai dengan tabel di atas adalah . . . .a. Jumlah pengangguran lulusan universitas lebih banyak daripada lulusan diploma.b. Jumlah pengangguran lulusan SLTA tahun2008 dan 2009 lebih sedikit daripada lulusan SLTA tahun 2010 dan

2011. c. Jumlah pengangguran lulusan SLTA setiap tahunnya menurun.d. Jumlah pengangguran lulusan SLTP setiap tahun menurun.e. Jumlah lulusan diploma setiap tahun menurun.

34. Kata itu pada kalimat kesatu paragraf kedua merujuk pada . . . .a. Empat ratus lima puluh ribu penduduk Indonesia membuka usaha diri.b. 0,81% penduduk Indonesia menjadi pengusaha.c. Empat ratus ribu penduduk Indonesia membuka usaha sendiri.d. Setipa pengusaha mampu merekrut rata- rata lima puluh pekerja.e. Jumlah angkatan kerja Indonesia 100 juta orang.

35. Paragraf yang paling tepat untuk melengkapi teks di atas adalah . . . .

Page 13: SBMPTN REVISI

a. Tabel di atas menunjukkan bahwa angka pengangguran dilploma dan SLTA paling tinggi. Mereka langsung masuk ke bursa tenaga kerja begitu tamat sekolah. Hal itu disebabkan tidak banyak tamatan yang dapat melanjutkan ke jenjang pendidikan di atasnya.

b. Tabel di atas menunjukkan bahwa angka pengangguran dilploma dan sarjan paling banyak jumlahnya. untuk mengantisipasi semakin bertambahnya jumlah itu, pendidikan kewirausahaan harus dilakukan di mana- mana.

c. Tabel di atas menunjukkan bahwa pengangguran tamatan universitas jumlahnya paling banyak. Untuk mengantisipasi semakin bertambahnya jumlah itu,pendidikan kewirausahaan harus dilakukan terutama melalui politeknik.

d. Dari tabel di atas diketahui bahwa pengangguran tamatan SLTA paling banyak jumlahnya. Hal itu disebabkan tidak banyak tamatan SLTA yang melanjutkan studi ke jenjang penididikan di atasnya, baik diploma maupun universitas.

e. Dari tabel di atas diketahui bahwa jumlah tamatan SLTA paling banyak. Jumlah itu harus ditampung dijenjang pendidikan di atasnya, yaitu diploma dan universitas. Hal tersebut dilakukan untuk mengurangi angka pengangguran.

36. Bacalah dengan cermat paragraf berikut!Kemunculan pola- pola lingkaran geometris di ladang pertanian atau crop cicke selalu . . . dengan kehadiran

alien di tempat itu denga mengendarai UFO. Dugaan tersebut didasarkan keyakinan bahwa manusia tidak mampu . . . pola serumit itu dalam waktu singkat dengan hasil yang hampir sempurna. Sesuai namanya, UFO atau unidentified flying object adalah objek terbang yang tidak . . .. Bentuknya bermacam- macam, ada yang seperti titik cahaya yang diam sejenak dan menghilang cepat, ada yang seperti piring terbang. UFO selalu dikaitkan dengan alien, sang makhluk luar angkasa yang dalam film- film fiksi ilmiah . . . sebagai sosok kecil, berkepala gundul, teling lebar, dan berwarna hijau. Ia digambarkan memiliki kekuatan super, termasuk . . . dirinya sama persis denga makhluk bumi.Urutan kata yang paling tepat untuk melengkapi bagian rumpang pada teks tersebut adalah . . . .a. Dihubungkan, membentuk, dikenal, ditampilkan, membuatb. Dihubungkan, membentuk, dikenali, digambarkan, menciptac. Dikaitkan, menghasilkan, dikenal, dimunculkan, membentukd. Dikaitkan, membuat, dikenali, digambarkan, mengubahe. Disertai, membuat, dikenali, ditampilkan, memunculkan

37. Bacalah dengancermat paragraf berikut!Walaupun wajib belajar pendidikan dasar sembilan tahun sudah dicanangkan dalam kurun waktu yang

relatif lama, penuntasannya masih belum tercapai. Banyak masalah yang timbul dalam pelaksanaan wajib belajar sembilan tahun, teruatam di daerah pedesaan dan daerah pegunungan atau terpencil. Penyebab ketidaktuntasan wajib belajar dapat diidentifikasi sesuai dengan kondisi wilayah dan masyrakatnya. Dari sejumlah hasil penelitian ditemukan bahwa penyebabnya adalah 1) masyarakat memiliki kondisi ekonomi yang lemah, 2) sosial budaya masyarakat yang kurang mendukung, 3) kurangnya sarana pendidikn, 4) rendahnya kualitas dan dedikasi guru, 5) letak geografis yang sulit dijangkau, 6) keterbatasan informasi, 7) persepsi masyarakat yang menganggap kurang pentingnya pendidikan bagi dirinya sendiri. Kenyataan itu diperkuat oleh hasil penelitian pada awal dicanangkannya wajib belajar enam tahun.Kata itu yang dicetak miring pada teks terebut merujuk pada . . . .a. Banyaknya masalah di daerah terpencilb. Penuntasan wajib belajar belum tercapaic. Banyaknya masalah pelaksanaan wajib belajard. Ketidaktuntasan wajib belajare. Penyebab ketidaktuntasan wajib belajar

38. Bacalah dengan cermat teks berikut!Teks I

Sungguh pun sudah ada program yang namanya Bantuan Operasional Sekolah (BOS), masih banyak anak Indonesia yang kesulitan mengakses pendidikan di luar sekolah dasar dan menengah. Berdasarkan angka resmi yang dikelurakan pemerintah, angka putus sekolah untuk tingkat SD dan SMP sekitar 768.960 orang yang terdiri atas 527.850 siswa SD dan 241.110 siswa SMP. Belum lagi, masih ada sekitar 8,3 juta orang Indonesia yang masih buta huruf.Teks II

Salah satu penyebab lancarnya penuntasan wajib belajar sembilan tahun di Kabupaten Bandung adalah adanya dukungan bOS yang dikucurkan sejak tahun 2005. Memasuki tahun 2009 program BOS mengalami perubahan tujuan, pendekatan, dan orientasi dari yang semula untuk perluasan akses menjadi peningkatan kualitas. Jumlah siswa yang menerima dana BOS di Kabupaten Bandung tercatat 511.228 siswa. Masing- masing SD Negeri sederajat 9.122 siswa, SMP Negeri sederajat 69.289 siswa, SMP swasta sederajat53.116 siswaPernyataan yang paling sesuai dengan kdeua teks tersebut adalah . . .a. Kedua teks tersebut mengungkap kegagalan BOS.b. Kedua teks tersebut mengungkap keberhasilan BOS.c. Teks I mengungkap kegagalan BOS, sedangkan teks II mengungkap keberhasilan BOS.d. Teks I mengungkap keberhasilan BOS, sedangkan teks II mengungkap kegagalan BOS.e. Teks I mengungkap akibat kegagalan BOS, sedangkan teks II menungkap penyebab keberhasilan BOS.

39. Seseorang akan mengembangkan karangan imiah yang bertema pembiasaan cuci tangan. Berdasarkan tema tersebut, kerangkan isi karang yang paling runtut berdasarkan pola umum- khusus adalah . . . .

Page 14: SBMPTN REVISI

a. Pentingnya pembiasaan cuci tangan, manfaat pembiasaan cuci tangan, bentuk pembiasaan cuci tangan, langkah pembiasaan cuci tangan.

b. Manfaat pembiasaan cuci tangan, pentingnya pembiasaan cuci tangan, sasaran pembiasaan cuci tangan, langkah pembiasaan cuci tangan, bentuk pembiasaan cuci tangan.

c. Pentingnya pembiasaan cuci tangan, sasaran pembiasaan cuci tangan, manfaat pembiasaan cuci tangan, manfaat pembiasaan cuci tangan, bentuk pembiasaan cuci tangan, langkah pembiasaan cuci tangan.

d. Sasaran pembiasaan cuci tangan, manfaat pembiasaan cuci tangan, pentningnya pembiasaan cuci tangan, bentuk pembiasaan cuci tangan, langkah pembiasaan cuci tangan.

e. Bentuk pembiasaan cuci tangan, pentingnya pembiasaan cuci tangan, langkah pembiasaan cuci tangan, sasaran pembiasaan cuci tangan, manfaat pembiasaan cuci tangan.

Teks berikut untuk soal nomor 40 s.d. 43. Bacalah dengan cermat!1) Laporan kegiatan analisis Tes Kompetensi Dasar mengatakan bahwa rerata kemampuan calistung

(membaca, menulis, berhitung) untuk seluruh kabupaten di propinsi DIY belum dapat memenuhi kriteria dan perlu ditingkatkan.2) hal ini beralasan jikamelihat hasil survei Direktorat Ketenagaan Depdiknas Tahun 2004 yang menyebutkan secara nasional 65,29% dari sebanyak 25.238 guru SD tidak menguasai dengan baik materi pelajaran ( Mundilarto, 2005:8). 3) Hal ini memiliki pengeruh terhadap kualitas soal yang digunakan dalam Ujian Sekolah Berstandar Nasioanal Pendidikan (UASBN), yakni komposisi butir soal UASBN sebanyak 25% diterapkan oleh Badan Standar Pendidikan (BSNP) yang berlaku secara nasional, serta 75% soal ditetapkan oleh penyelanggara UASBN tingkat propinsi (Peraturan Mendiknas 2007). 4) Padahal, angka literasi telah dijadikan salah satu indikator oleh UNDP (United Nations Development Program) untuk mengukur kualitas bangsa. 5) Tinggi rendahnya angka literasi menentukan tinggi rendahnya Indeks Pembangunan Manusia atau HDI ( Human Development Index) dan tinggi rendanhya HDI menentukan kualitas bangsa.

6) Dalam publikasi UNDP terakhir,”Human Development Report 2003” Indonesia ditempatkan di peringkat 112 dari 174 negara dalam hal kualitas bangsa. 7) Ditambah hasil survei pengukuran dan penilaian pendidikan oleh Trends in Internatioanal Mathematicd and Science Study (TIMSS) 1999 terhadap 38 negara, nilai matematika dan IPA anak- anak Indonesia juga sangat rendah.8) Hasil pengukuran daya serap kurikulum siswa secara nasional oleh Direktorat Pendidikan TK dan SD tahun 2000/ 2001 menunjukkan bahwa rata- rata daya serap kurikulum secaranasional masih rendah, yaitu 5,1 untuk lima mata pelajaran (Siregar, 2005:5)

40. Pada kalimat manakah kesalahan penggunaan ejaan (tanda baca, penulisan huruf, dan penulisan kata ditemukan?a. 1,3, dan 5 d. 2,6, dan 8b. 1,2 dan 4 e. 3,7, dan 8c. 3,5, dan 7

41. Jika Anda mengutip pendapat Siregar tanpa membaca sumber aslinya, penulisan kutipan bertingkat yang paling benar adalah . . . .a. Menurut Siregar (dalam Narbito, 2010:12), rerata daya serap kurikulum secara nasioanal masih rendah, yaitu

5,1 untuk lima mata pelajaran yang dujikanb. “Rerata daya serap kurikulum secara nasional masih rendah, yaitu 5,1 untuk lima mata pelajaran “ (Siregar,

2005:5)c. Siregar (dalam Reka Narbito, 2010) menyatakan “Rerata daya serap kurikulum secara nasional masih rendah,

yaitu 5,1 untuk lima mata pelajaran”.d. Dalam buku Narbito, Siregar (2005:12) menyatakan bahwa rerata daya serap kurikulum secara nasional rendah

untuk lima mata pelajaran yang diujikane. Siregar (2005) menyatakan “Rearata daya serap kurikulum secara nasional masih rendah, yaitu 5,1 untuk lima

mata pelajaran” (Narbito, 2010)42. Rangkuman teks di atas yang paling tepat adalah . . . .

a. Kompetensi calistung siswa SD di DIY rendah, sementara kompetensi tersebut merupakan satu- satunya indikator keberhasilan kualitas suatu bangsa.

b. Calistung merupakan kompetensi dasar siswa SD di DIY yang perlu ditingkatkan pencapaiannya sebab itulah indikator HDI yang harus dicapai.

c. Calistung merupakan kompetensi dasar siswa SD se- DIY yang masih perlu ditingkatkan pencapaiannya sebab itulah yang menentukan kualitas bangsa.

d. Kompetensi calistung siswa SD di DIY masih rendah sehingga memprihatinkan semua pihak mengingat indikator HDI yang menentukan kualitas bangsa.

e. Kompetensi calistung siswa SD di DIY rendah, sementara kompetensi tersebut merupakan salah satu indikator HDI yang menentukan kualitas bangsa.

43. Pola paragraf pertama teks tersebut adalah . . . .a. Kalimat utama- kalimat penjelas- kalimat penjelas, kalimat penjelas, kalimat penjelas.b. Kalimat penjelas, kalimat utama, kalimat penjelas, kalimat penjelas, kalimat penjelas.c. Kalimat penjelas, kalimat penjelas, kalimat penjelas, kalimat penjelas, kalimat utamad. Kalimat utama, kalimat penjelas, kalimat penjelas, kalimat penjelas, kalimat utama.e. Kalimat utama, kalimat utama, kalimat penjelas, kalimat penjelas, kalimat penjelas.Teks berikut untuk soal nomor 44 s.d 47. Bacalah dengan cermat!

Pendidikan adalah serangkaian tugas ataupun misi yang diemban dan dilaksanakan oleh pendidikan. Dari pengertian ini dapat dijelskan bahwa sesungguhnya kegiatan atau praktik pendidikan di manapun bukanlah kegiatan tanpa makna dan tujuan yang jelas. Kegiatan pendidikan tersirat suatu tugas atau misi yang harus diwujudkan. Oleh karena itu, para pendidik, pengelola pendidikan, dan pihak- pihak yang terlibat dalam kegiatan

Page 15: SBMPTN REVISI

pendidikan seharusnya selalu menyadari akan tugas atau misi kegiatan pendidikan yang dialkasanakan atau yang dikelolanya.

Dari segi kultural dan sosial, fungsi pendidikan adalah menumbuhkan nilai- nilai insaniah; serta menyiapkan tenaga kerja produktif. Sementara itu, Bock mengatakan bahwa peran pendidikan adalah memasysrakatkan ideologi dan nilai- nilai sosio- kultural bangsa; mempersiapkan tenaga kerja untuk mendorong perubahan sosial; serta meratakan kesempatan dan pendapatan. Dari sumber tersebut jelas, bahwa kegiatan atau praktik pendidikan memiliki misi atau tugas bukan hanya menjadikan peserta didik tumbuh dan berkembang potensi kemampuannya, melainkan juga (1) menjga dan melestarikan nilai- nilai yang dianggap baik dan benar oleh masysrakat dan bangsa serta (2) menyiapkan peserta didik menjadi tenaga kerja produktif. Di pihak lain, fungsi pendidikan nasional menurut Undanf- Undang Nomor 20 Tahun 2003 adalah mengembangkan kemampuan dan membentuk watak serta peradaban bangsa yang bermartabat dalam rangka mencerdaskan bangsa . . . .

44. Simpulan yang paling tepat untuk teks tersebut adalah . . . .a. Tujuan pendididkan nasional di Indonesia telah mencakuo segi kuktural dan sosial dan menjangkau semua

lapisan masysrakat dalam hal mencerdaskan bangsa serta menyiapkan tenaga kerja produktif.b. Pendidikan bertugas menjadikan peserta didik tumbuh dan berkembang potensinya, melestarikan nilai- nilai

yang baik dan benar serta menyiapkan tenaga produktif.c. Fungsi pendidikan nasional menurut Undang- Undang Nomor 20 Tahun 2003 selaras dengan pendaoat Bock

yang menagatakan bahwa aperan pendidikan adalah memasyrskatkan ideologi.d. Kegiatan pendidikan memiliki misi bukan hanya menjadikan peserta didik tumbuh dan berkembang potensinya,

melainkan juga menjaga dan melestarikan nilai- nilai yang dianggap baik dan benar.e. Pendidikan nasional menurut Undang- Undang Nomor 20 Tahun 2003 berfungsi mengembangkan kemampuan

membentuk watak serta peradaban bangsa yang bermartabat dalam rangka mencerdaskan bangsa.45. Suntungan yang paling tepat untuk kalimat kedua dan ketiga paragraf pertama teks tersebut adalah . . . .

a. Dari pengertian tersebut, dapat dijelaskan kegiatan atau praktik pendidikan dimanapun bukanlah tanpa makna dan tujuan yang jelas. Dari kegiatan pendidikan tersirat suatu tugas atau misi yang harus diwujudkan.

b. Pengertian ini dapat dijelaskan bahwa kegiatan pendidikan tidaklah kegiatan tanpa makna dan tujuan yang jelas. Melalui kegiatan pendidikan tersirat suatu tugas atau misi yang harus diwujudkan.

c. Pengertian ini dapat dijelaskan bahwa pendidikan tidaklah kegiatan tanpa mekna dan tujuan yang jelas. Melalui kegiatan pendidikan tersirat tugas atau misi yang harus diwujudkan.

d. Dari pengertia itu, dapat dijelaskan kegiatan atau praktik pendidikan bukanlah tanpa makna dan tujuan yang jelas. Dalam kegiatan pendidikan tersirat suatu tugas atau misi yang harus diwujudkan.

e. Dari pengertian itu dapat dijelaskan bahwa kegiatan atau praktik pendidikan di manapun bukanlah tanpa makna dan tujuan yang jelas. Dalam kegiatan pendidikan tersirat auatu tugas atau misi yang harus diwujudkan.

46. Agar menjadi paragraf yang baik, kalimat penutup yang paling sesuai untuk paragraf kedua adalah . . . .a. Dengan demikian, fungsi pendidikan selain untuk mencrdaskan bangsa, menyiapkan tenaga kerja produktif juga

membentuk watak.b. Dengan demikian, fungsi pendidikan adalah menjadikan siswa terampil dan siap menjdai tenaga kerja produktif

dan menjunjung tinggi nama bangsa.c. Oleh karena itu, tugas pendidikan membentuk siswa menjadi warga negara yang berkepribadian luhur,

beriman, dam bertakwa.d. Karena itulah, fungsi utama pendidikan adalah membentuk siswa yang cerdas dan terampil dalam menghadapi

semua permasalahan.e. Oleh sebab itu, sebagai pendidik, gurui harus selalu siap berkorban untuk membentuk anak bangsa menjadi

manusia berguna.47. Gagasan utama paragraf kedua pada teks tersebut adalah . . . .

a. Fungsi pendidikan adalah memasyarakatkan ideologi dan nilai sosio- kultural, mempersiapkan tenaga kerja.b. Fungsi pendidikan dari segi kultural dan sosial adalah menumbhukan kreativitas subjek didik serta menyiapkan

tenaga kerja produktif.c. Fungsi pendidikan adalah menumbuhkan kreativitas, nelestarikan nilai- nilai, dan menyiapkan tenaga kerja.d. Fungsi pendidikan adalah melestarikan nilai yang baik dan benar serta menyiapkan peserta didik menjadi

tenaga kerja.e. Fungsi pendidikan adalahmelestarikan nilai- nilai dan menyiapkan peserta didik menjadi tenaga kerja produktif.

48. Bacalah dengan cermat paragraf beariktu!Sejumlah pengusaha sepatu kecil menengah di Kabupaten Mojokerto mengalami kesulitan menghadapi

aturan perdagangan bebas atau free trade agreement (FTA) ASEAN- Cina .merka mengaku merugi sejak isu itu digulirkan pada awal Januari lalu. Para pedagang mengalami dampak negatif aturan itu. pada sektor usaha kecil sepatu, misalnya, usaha merugu sampai 50%. “Orderan berkurang 50%. Artinya, produksi kami juga berkurang sebesar itu,” kata Budi Utomo, anggota Gabungan Pengusaha Sepatu (GPS) Kabupaten Mojokerto. Sepinya order terjadi karena sebagian besar pelanggan tidak melakukan pemesanan karena menunggu masuknya produk impor lebih murah daripada harga produk lokal.Pernyataan berikut yang tidak sesuai dengan isi paragraf tersebut adalah . . . .a. FTA ASEAN- Cina membuat panik sejumlah pengusaha sepatu kecil menengah di Kabupaten Mojokerto.b. Para pelanggan pengusaha di Kabupaten Mojokerto lebih memilih produk sepatu impor daripada membeli

sepatu lokal.c. Sejak Januari lalu, kerugian akibat FTA ASEAN- Cina dirasakan para pengusaha sepatu kecil menengah

Kabupaten Mojokerto.

Page 16: SBMPTN REVISI

d. Akibat FTA ASEAN- Cina kerugian para pengusaha kecil di Kabupaten Mjokerto mencapai sekitar 50%.e. Banyak pelanggan pengusaha sepatu di Kabupaten Mojokerto menghentikan permintaan karena menunggu

pelaksanaan FTA ASEAN- Cina.49. Bacalah dengan cermat teks berikut!

Lidah lalat buah ternyata membantu memberikan jawaban atas pertanyaan mengenai kebiasaan makan manusia. Sebuah studi yang dilakukan tim dari Texas A & M University meangungkap bahwa lidah lalat itu juga menawarkan jalan baru untuk menangani obesitas. Paul Hardin dkk. Menelitiorgan perasa pada dorosphila yang memicu hasarat lalat buah untuk makan atau sebaliknya. Mereka menemukan bahwa jam tubuh harian serangga itu menemukan perilaku makan dan sensivitas rasa. Ini kemingkinan juga berlaku juga pada manusia. Riset menunjukkan bahwa sensistivitas tinggi terhadap gula terjadi pada siang hari dan berkurang pada malam hari. Jika jam itu dihilangkan, lalat akan melahap lebih banyak makanan. Jadi, jam ini tampaknya menekan keinginan untuk makan pada waktu tertentu.Rangkuman yang tepat untu paragraf di atas adalah . . . .a. Riset tim Texas A & M University menunjukkan sel sensor lidah lalat buah bisa mengendalikan waktu makan dan

jumlah makanan serta sangat berdampak terhadap nafsu makan serangga.b. Penelitian dari Texas A & M University menunjukkan bahwa lidah lalat buah ternyata memberikan jawaban

besar atas pertanyaan mengenai perilaku kebiasaan makan heawan tersebut yang mungkin juga berlaku bagi manusia.

c. Studi yang dilakukan tim dari Texas A & M University mengungkap bahwa penelitian terhadapa lidah lalat buah dapat menjadi solusi untuk mengendalikan nafsu makan pada serangga.

d. Penelitian yang dilakukan oleh tim dari Texas A & M University menunjukkan bahwa lidah lalat buah dan jam tubuh hewan tersebut menjadi salah satu faktor pengenadali perilaku makan heawan tersebut yang mungkin berlaku pada manusia.

e. Hasil penelitian terhadap laidah lalat buah yang terkait dengan pengendalian waktu makan dan jumlah makanan yang dikonsumsi dapat menawarkan jalan baru untuk menangani obesitas pada menusia.

50. Bacalah dengan cermat paragraf berikut!Fisika plasma merupakan salah satu bidang fisika yang . . . gas terionisai, yang terkenal sebagai plasma.

Dalam fisika dan kimia, plasma adalah . . . fase gas berenergi, yang sering ditunjuk sebagai “keadaan benda keempat”, yang beberapa atau semua elektron di orbit atom terluar telah .. . dari atom molekul. Hasilnya adalah sebuah koleksi ion dan elektro yang tidak lagi . . . satu dengan yang lain.Urutan kata yang tepat untuk melengkapi bagian yang rumpang pada teks di atas adalah . . . a. mempelajari, keadaan, terpisah, terkait d. menguraikan, sifat, terikat, terpisahb. membahas, keadaan, terdiri, terpisah e. meliputi, keadaan, terurai, terpaduc. menyangkut, sifat, terpisah, menyatu

51. Bacalah dengan cermat paragraf berikut!Peningkatan kesejahteraan masyarakat merupakan tanggung jawab bersama antara pemerintah dan

masyarakat. Baik pria maupun wanita, hendaknya selalu meningkatkan rasa tanggung mereka terhadap kesejahteraan bersama. Kader- kader wanita diharapkan juga peduli terhadap kesejahteraan masyarakat pada umumnya. Banyak kaum wanita yang dipaksa bekerja karena suami mereka terkena PHK. Untuk itu, pemerintah perlu memperhatikan nasib kaum wanita yang suami mereka terkena PHK.Ide pokok paragraf di atas adalah . . . .a. Peningkatan kesejahteraan masyarakat perlu dilakukan secara bersama, antara pria dan wanita.b. Kesejahteraan keluarga korban PHK menjadi tanggung jawab pemerintah.c. Kader- kader wanita diharapkan juga peduli terhadap kesejahteraan masyarakat pada umumnya.d. Pemerintah dan masyarakat bertanggung jawab terhadap kesejahteraan masyarakat.e. Demi kesejahteraan keluarga, kaum wanita yang suami mereka di- PHK terpaksa bekerja.

52. Bacalah dengan cermat paragraf berikut!1) Para peneliti mengemukakan bahwa area otak untuk bernyanyi berbeda dengan area otak untuk

berbicara. 2) Mereka menyajikan temuan mereka terebut pada pertemuan American Associatioan for the Advancement of Science di San Diego. 3) Jika pusat berbicara seseorang rusak oleh stroke, ia dapat belajar menggunakan pusat bernyanyinya. 4) Dalam uji coba klinis, para peneliti menunjukkan cara otak merespon terapi intonasi melodi. 5) Gottried Schalug, profesor neurologi Beth Israel Deaconess Medical Center dan Harvard Medical School di Boston Amerika Serikat memimpin uji coba tersebut. 6) Penelitian itu menemukan bahwa pasien stroke dengan kerusakann otak membuat pasien tidak bisa bicara, tetapi bisa bernyanyi.Kalimat utama paragraf tersebut adalah . . . a. Kalimat 1 b. kalimat 3 c. kalimat 4 d. kalimat 5 e. kalimat 6

53. Bacalah dengan cermat paragraf berikut!Affandi adalah putra R. Koesoema, seorang mantri ukur di pabrik gula Ciledug, Cirebon. Affandi memiliki

pendidikan formal yang cukup tinggi. Bagi orang- orang segenarisnya, hanya segelintir anak negeri saja yang memperoleh pendidikan HIS, MULO, dan AMS. Akan tetapi, bakat seni lukisannya yang sangat kuat mengalahkan disiplin ilmu lain dalam kehidupannya. Bakat itu menjadikan nama Affandi tenar, sama dengan nama- nama tokoh atau pemuka bidang lainnya. Sebelum menjadi pelukis, Affandi pernah menjadi guru, tukang sobek karcis, dan pembuat gambar reklame bioskpo di salah satu gedung bioskop di Bandung. Pekerjaan tersebut tidak lama ia geluti karena Affandi lebih tertarik pada seni lukis. Sekitar tahun30-an, Affandi bergabung dalam Kelompok Lima Bandung,yaitu kelompok lima pelukis Bandung. Kelompok ini memiliki andilbesar dalam perkembangan seni rupa Indonesia.

Page 17: SBMPTN REVISI

Pernyataan berikut yang sesuai dengan isi teks di atas adalah. . . .a. Pendidikan informal Affandi (di HIS, MULO, dan AMS) cukup tinggi pada masa generasinya.b. Kelompok Lima Bandung didirikan Affandi untuk mengembangkan seni rupa Indonesia.c. Melukis menjadi pekerjaan paling lama digeluti Affandi dalam hidupnya.d. Profesi Affandi dirintis sejak awal memulai pendidikan formalnya di HIS, MULO, dan AMS. e. Affandi menjadi pelukis bersama pelukis terkenal, yaitu R. Koesoema.

54. Bacalah dengan cermat tabel berikut!Tabel Produksi Jagung Kecamatan Sukamaju

Tahun Daerah A Daerah B Daerah C Daerah D2006 375.000 375.000 385.000 340.0002007 380.000 375.000 395.000 349.0002008 385.000 386.000 405.000 360.000

Pernytaan yang sesuai dengan isi tabel tersebut adalah . . . .a. Persentase kenaikan produksi jagung daerah C paling tinggi.b. Produksi jagung setiap daerah selalu meningkat setiap bulan.c. Persentase kenaikan produksi jagung daerah D paling tinggi.d. Persentase kenaikan produksi jagung daerah B paling rendah.e. Persentase kenaikan produksi jagung daerah D paling rendah.

55. Bacalah dengan cermat paragraf berikut!Dalam ilmu ekonomi, inflasi adalah suatu proses meningkatnya harga- harga secara umum dan terus-

menerus berkaitan dengan mekanisme pasar. Istilah infalsi juga berarti peningkatan persediaan uang yang menyebabkan kenaikan harga berlangsung secara terus- menerus dan saling memengaruhi. Agar menjadi paragraf yang baik, kalimat yang sesuai dengan paragraf tersebut adalah . . . a. Dengan demikian, adadua cara untuk mengukur tingkat infalsi, yaitu CPI dan GDO deflator.b. Akibatnya, untuk mengukur tingkat inflasi diperlukan dua cara, yaitu CPI dan GDP deflator.c. Jadi, ada dua cara untuk mengukur tingkat inflasi, yaitu CPI dan GDP deflator.d. Dalam hal ini, ada dua cara untuk mengukur tingkat inflasi, yaitu CPI dan GDP deflator.e. Oleh karena itu, ada dua cara untuk mengukur tingkat inflasi, yaitu CPI dan GDP deflator.

56. Bacalah dengan cermat paragraf berikut!Belajar di Luar Negeri tidak harus selalu ke Perguruan Tinggi di Amrika Serikat, atau ke negara- negara di

Eropa karena kualitaas perguruan tinggi tidak ditentukan oleh lokasi tetapi oleh academic performance.Perbaikan ejaan kalimat di atas yang tepat adalah . . . .a. Belajar di luar negeri tidak harus selalu ke Perguruan Tinggi di Amerika Serikat, atau ke negara- negara di Eropa

karena kualitaas perguruan tinggi tidak ditentukan oleh lokasi tetapi oleh academic performance.b. Belajar di luar negeri tidak harus selalu ke perguruan tinggi di Amerika Serikat, atau ke negara- negara di Eropa

karena kualitaas perguruan tinggi tidak ditentukan oleh lokasi, tetapi oleh academic performance.c. Belajar di luar negeri tidak harus selalu ke perguruan tinggi di Amerika Serikat, atau ke negara- negara di Eropa

karena kualitaas perguruan tinggi tidak ditentukan oleh lokasi tetapi oleh academic performance.d. Belajar di Luar Negeri tidak harus selalu ke Perguruan Tinggi di Amerika Serikat, atau ke negara- negara di Eropa

karena kualitaas perguruan tinggi tidak ditentukan oleh lokasi tetapi oleh academic performance.e. Belajar di luar negeri tidak harus selalu ke negara- negara di Eropa karena kualitaas perguruan tinggi tidak

ditentukan oleh lokasi tetapi oleh academic performance.57. Bacalah dengan cermat data berikut!

Berikut ini adalah identitas buku yang dapat digunakan sebagai sumber untuk menulis karanganJudul Buku Penulis Tahun Penerbit KotaPotret Kehidupan Penduduk Miskin B. Setiawan 2009 Jaya Maya YogyakartaIde Genius Lahirnya BLT di Indonesia Jalaludin Rakhmat 2009 Remaja Jaya BandungPengentasan Kemiskinan di Indonesia Prof. Arif Muntaha 2009 Harapan Jaya YogyakartaMiskin Harta Kaya Hati Harsono Suwardi 2010 Sinar Harapan Surabaya

Daftar pustaka yang relevan dan tepat untuk laporan penelitian berjudul Pengentasan Kemsikinan di Indonesia adalah . . . .a. Arif Muntaha. (2009). Pengentasan Kemiskinan di indonesia. Yogyakarta: Harapan Jaya.

B.Setiawan. (2009). Potret Kehidupan Penduduk Miskin. Yogyakarta: Jaya Maya. Harsono Suwardi.(2010). Miskin Harta Kaya Hati. Surabaya: Sinar Harapan. Jalaludin Rakhmat. (2009). Ide Genius LahirnyaBLT di Indonesia. Bandung: Remaja Jaya.

b. Muntaha, A. 2009. Pengentasan Kemiskinan di Indonesia. Yogyakarta: Harapan Jaya.Setiawan, B. 2009. Potret Kehidupan Penduduk Miskin. Yogyakarta: Jaya Maya. Suwardi, Harsono.2010. Miskin Harta Kaya Hati. Surabaya: Sinar Harapan. Jalaludin Rakhmat. 2009. Ide Genius Lahirnya BLT di Indonesia. Bandung: Remaja Jaya.

c. Arif Muntaha.2009. Pengentasan Kemiskinan di indonesia. Yogyakarta: Harapan Jaya.B.Setiawan. 2009. Potret Kehidupan Penduduk Miskin. Yogyakarta: Jaya Maya. Jalaludin Rakhmat. 2009. Ide Genius LahirnyaBLT di Indonesia. Bandung: Remaja Jaya. Harsono Suwardi.2010. Miskin Harta Kaya Hati. Surabaya: Sinar Harapan.

d. Muntaha, A. (2009). Pengentasan Kemiskinan di indonesia. Yogyakarta: Harapan Jaya. Rakhmat, Jalaludin. (2009). Ide Genius LahirnyaBLT di Indonesia. Bandung: Remaja Jaya. Suwardi, Harsono. (2010). Miskin Harta Kaya Hati. Surabaya: Sinar Harapan.

Page 18: SBMPTN REVISI

e. Muntaha, A. 2009. Pengentasan Kemiskinan di Indonesia. Yogyakarta: Harapan Jaya. Rakhmat, J. 2009. Ide Genius Lahirnya BLT di Indonesia. Bandung: Remaja Jaya. Setiawan, B. 2009. Potret Kehidupan Penduduk Miskin. Yogyakarta: Jaya Maya.

58. Bacalah dengan cermat teks berikut!Pertumbuhan penggunaan internet yang pesat sekali di Indonesia telah diakui membawa pengaruh positif

dalam berbagai macam hal. Namun, masih banyak yang terlupakan, di sisi lain internet juga berpotensi memberikan dampak buruk, khususnya kepada golongan anak- anak. Anak- anak dan remaja menjadi golongan yang rentan terhadap praktik kejahatan siber, seperti pencabulan.Paragraf tersebut dapat diperbaiki dengan cara berikut ini, kecuali . . . .a. Kata diakui dan macam dihilangkanb. Tanda koma sebelum kata seperti dihilangkanc. Tanda baca titik diletakkan setelah kata terlupakand. Kata hubung namun diganti akan tetapie. Kata kepada dihilangkan.

59. Bacalah dengan cermat kalimat berikut!Prof. DR. Muhammad Nuh, Menteri Pendidikan Nasional mengatakan, hasil ujian Nasional Tahun ini belum

dapat digunakan sebagai penentu penerimaan mahasiswa baru melalui SNMPTN.Kalimat tidak efektif tersebut dapat diperbaiki dengan cara berikut, kecuali . . .a. Huruf R pada singkatan DR. dituliskan dengan huruf kecilb. Kata bahwa ditambahkan setelah kata mengatakanc. Tanda baca koma setelah kata mengatakan dihilangkand. Kata Ujian Nasional ditulis ujian nasionale. Tanda baca koma setelah kata Nuh dihilangkan

60. Bacalah dengan cermat ilustrasi berikut!Dalam majalah Bunda yang diterbitkan tahun 2010, pada halaman 37, terdapat kutipan pendapat Sugiyarto dalam artikel karya Agra Mashudi sebagai berikut:Orang gemuk dan anak- anak usia sekolah jauh lebih rentan terserang sakit hingga meninggal akibat virus flu babi H1N1 (Sugiyarto, 2008: 11).Jika Ikang mengutip pendapat Sugiyarto tanpa membaca bukunya, kutipan yang benar adalah . . . a. Menurut Sugiyarto (2008: 11) orang gemuk dan anak- anak usia sekolah jauh lebih rentan terserang sakit hingga

meninggal akibat virus flu babi H1N1 (dalam Mashudi, 2010).b. Sugiyarto (2008) dalam (Mashudi,2010) mengatakan bahwa orang gemuk dan anak- anak usia sekolah jauh

lebih rentan terserang sakit hingga meninggal akibat virus flu babi H1N1.c. Sugiyarto (dalam Mashudi, 2010) menyatakan bahwa orang gemuk dan anak- anak usia sekolah jauh lebih

rentan terserang sakit hingga meninggal akibat virus flu babi H1N1.d. Orang gemuk dan anak- anak usia sekolah jauh lebih rentan terserang sakit hingga meninggal akibat virus flu

babi H1N1 ( Sugiyarto, 2008 dalam Mashudi, 2010).e. Menurut Sugiyarto (2008:11 dalam Mashudi . 2010: 37) menayatakan bahwa “orang gemuk dan anak- anak usia

sekolah jauh lebih rentan terserang sakit hingga meninggal akibat virus flu babi H1N1”61. Untuk mengembangkan karangan dengan topik pembinaan koperasi usaha kecil dan menengah, kerangka karangan

yang baik adalah . . . a. Pentingnya pembinaan koperasi usaha kecil dan menengah, langkah pembinaan koperasi usaha kecil dan

menengah, sasaran pembinaan koperasi usaha kecil dan menengah, bentuk pembinaan koperasi usaha kecil dan menengah.

b. Bentuk pembinaan koperasi usaha kecil dan menengah, sasaran pembinaan koperasi usaha kecil dan menengah,pentingnya pembinaan koperasi usaha kecil dan menengah,langkah pembinaan koperasi usaha kecil dan menengah.

c. Pentingnya pembinaan koperasi usaha kecil dan menengah,langkah pembinaan koperasi usaha kecil dan menengah, bentuk pembinaan koperasi usaha kecil dan menengah, sasaran pembinaan koperasi usaha kecil dan menengah.

d. Pentingnya pembinaan koperasi usaha kecil dan menengah, sasaran pembinaan koperasi usaha kecil dan menengah, bentuk pembinaan koperasi usaha kecil dan menengah, langkah pembinaan koperasi usaha kecil dan menengah.

e. Sasaran pembinaan koperasi usaha kecil dan menengah, pentingnya pembinaan koperasi usaha kecil dan menengah, langkah pembinaan koperasi usaha kecil dan menengah, bentuk pembinaan koperasi usaha kecil dan menengah.

62. Bacalah dengan cermat paragraf berikut!1)Sekelompok anjing laut utara ( Mirounga angustirostis) sedang berganti kulit di sebuah pantai di dekat

San Simoen, California. 2) Pantai ini sudah dijadikan tempat berkumpulnya sekelompok anjing laut utara. 3) Di tempat ini pada musim panas, anjing laut mengalami pergantian kulit besar- besaran yang berlangsung sekitar empat bulan. 4) Selama sebulan, mereka kehilangan bulu dan kulit. 5) Anjing laut utara menghangatkan badan mereka sewaktu musim berganti kulit dengan cara berkumpul di pantai sambil menunggu tumbuhnya kulit baru.Kalimat topik paragraf tersebut adalah kalimat nomor . . . .a. 1 b. 2 c. 3 d. 4 e. 5

63. Bacalah dengan cermat paragraf berikut!

Page 19: SBMPTN REVISI

Departemen Agama seharusnya snggup menyelenggarakan ibadah haji secara baik. Sesuai dengan undang-undang terbaru, departemen ini tetap menjadi regulator sekaligus operator, bahkan ikut menentukan anggota komisi pengawas. Peningkatan kualitas pelayanan diharapkan terjadi lewat pem- benahan manajemen. Keluhan mulai muncul ketika belum ada perbaikan yang berarti. Lihatlah, baru-baru ini 89 anggota jemaah calon haji dari Jakarta dilaporkan mengalami diare saat berada di Madinah. Penderitaan mereka diduga akibat makanan yang basi atau tidak memenuhi standar kesehatan. Insiden seperti ini tidak perlu terjadi seandainya perusahaan katering menjaga kualitas makanan yang disajikan. Kualitas pelayanan akan terjamin lagi jika tim kesehatan selalu mengecek dan mengawasi makanan untuk jemaah calon haji. ( Tempo, 211108). Pernyataan berikut merupakan opini dari teks di atas, kecuali…a. Jemaah calon haji Indonesia tidak memperoleh layanan maksimal dari Departemen Agama. b. Departemen Agama tidak bekerja secara profesional dalam penyelenggaraan ibadah haji. c. Manajemen Departemen Agama lemah sehingga tidak mampu meningkatkan pelayanan penyelenggaraan

ibadah haji. d. Tim kesehatan tidak mengecek dan mengawasi makanan sehingga jemaah calon haji makan nasi basi. e. Koordinasi tidak berjalan baik di antara komponen panitia penyelenggara ibadah haji di Departemen Agama.

64. Bacalah dengan cermat teks berikut!Saat ini diperlukan realisasi janji dari para pemimpin negeri ini, yakni peningkatan kualitas pendidikan

dengan anggaran hingga 20% dari total APBN dan APBD. Sayangnya, baru sekitar 44 kabupaten di Indonesia yang mengalokasikan anggaran pendidikan sesuai dengan ketentuan tersebut. Padahal, peningkatan kualitas pendidikan ini telah masuk dalam delapan prioritas pembangunan dalam program Susilo Bambang Yudhoyono. Dalam salah satu dari prioritas itu disebutkan dengan jelas bahwa peningkatan akses dan kualitas pendidikan dan kesehatan menjadi prioritas pembangunan indonesia. Namun, realisasinya masih sangat mengecewakan. (Koran Sindo, 211108). Pernyataan berikut yang tidak relevan dengan isi teks tersebut adalah…. a. Pemerintah menjanjikan anggaran pendidikan sebesar 20% dari APBN dan APBD. b. Alokasi anggaran pendidikan 20% baru dilakukan oleh 44 kabupaten. c. Janji pemerintah menetapkan anggaran pendidikan 20% belum ditepati. d. Pendidikan dan kesehatan merupakan prioritas utama program. e. Realisasi dari delapan prioritas pembangunan sangat mengecewakan.

65. Seseorang akan mengembangkan karangan yang bertema upaya mengatasi pergaulan bebas pada kaum remaja di perkoataan.Berdasarkan tema tersebut, kerangka karangan yang tepat adalah . . . .a. Masalah yang timbul akibat pergaulan bebas pada kaum remaja di perkoataan, pentingnya mengatasi pergaulan

bebas pada kaum remaja di perkoataan, langkah mengatasi pergaulan bebas, saran yang dapat dilakukan untuk mengatasi pergaulan bebas.

b. Pentingnya mengatasi pergaulan bebas pada kaum remaja di perkotaan, langkah mengatasi pergaulan bebas, langkah mengatasi pergaulan bebas, masalah yang timbul akibat pergaulan bebas pada kaum remaja di perkoataan, saran yang dapat dilakukan untuk mengatasi pergaulan bebas.

c. Langkah mengatasi pergaulan bebas, masalah yang timbul akibat pergaulan bebas pada kaum remaja di perkoatan, pentingnya mengatasi pergaulan bebas pada kaum remaja di perkoataan, saran yang dapat dilakukan untuk mengatasi pergaulan bebas.

d. Pentingnya mengatasi pergaulan bebas pada kaum remaja di perkoataan, langkah mengatasi pergaulan bebas, masalah yang timbul akibat pergaulan bebas pada kaum remaja di perkoatan, saran yang dapat dilakukan untuk mengatasi pergaulan bebas.

e. Pentingnya mengatasi pergaulan bebas pada kaum remaja di perkotaan, masalah yang timbul akibat pergaulan bebas pada kaum remaja di perkoataan, langkah mengatasi pergaulan bebas, saran yang dapat dilakukan untuk mengatasi pergaulan bebas.

66. Bacalah dengan cermat paragraf yang salah satu kalimatnya dirumpangkan berikut ini!…. Salah satu penyakit kulit adalah acne vulgaris. Tidak ada seorang pun di dunia ini yang tidak pernah

menderita penyakit ini. Acne vulgaris adalah penyakit peradangan folikel sebasea yang umumnya terjadi pada masa remaja dan dapat sembuh sendiri. Dalam masyarakat umum acne vulgaris biasa dikenal dengan istilah jerawat. Kalimat yang manakah yang tepat untuk mengisi bagian rumpang tersebut adalah… a. Acne vulgaris adalah penyakit kulit biasa. b. Acne vulgaris pernah dialami semua manusia. c. Penyakit kulit banyak jenis dan macamnya. d. Jerawat banyak jenis dan macamnya. e. Penyakit yang diderita remaja bermacam-macam.

67. Bacalah kutipan teks berikut dengan cermat!Pasar uang dan pasar modal Indonesia belum pulih. Rupiah masih terus mengalami tekanan mengikuti

penurunan sebagian mata uang Asia. Masalah minimnya pasokan dolar AS di pasar valas semakin mempersulit mata uang lokal ini. Pada penutupan perdangan valas tanggal 24 November 2008, rupiah melemah hingga 320 poin ke posisi 12. 320 per dolar AS. Rupiah bahkan sempat menembus 12.325 per dolar AS. Pelaku pasar cemas karena belum ada sentimen positif dari dalam negeri. Oleh karena itu, pasar lebih memilih memegang dolar AS dalam kondisi pasar global yang sedang rentan ini. Paragraf tersebut dikembangkan dengan menggunakan pola…

Page 20: SBMPTN REVISI

a. sebab-akibat b. akibat-sebab c. generalisai d. analogi e. silogisme 68. Bacalah dengan cermat teks berikut!

Tokoh- tokoh ternama, seperti Charlie Chaplin, Jawaharal Nehru, Gamal Abdul Naser, dan Bung Karno, pernah menginap di sini. Gaya seni Art Deco- nya yang unik seolah menjadi trademark hotel yang satu ini. Cobalah menyusuri Jalan Asia Afrika di Bandung. Di sana ada gedung yang memiliki lengkung bangunan unik berwarna keabuan. Keunikan tersebut seolah- olah tak terpisahkan dari sejara Kota Kembang yang pernah kental sekali aura Eropanya. Hotel yang kini disebut Savoy Hmann Bidakara Hotel ini selalu menjadi kebanggaan ‘ urang Bandung’. Selanjutnya di bawah pengelolaan Van Es, bangunan hotel Humann yang semula sempit dan sederhana diperluas serta dimodernisasi menjadi salah satu hotel terkemuka di Asia Tenggara. Gaya seni Art Deco yang melanda daratan Eropa pada tahun 1920- an ikut mewarnai bangunan Homann lama, dan hiasan interior, jendela kaca patri, ornamen dinding, kodel meubel, sampai kap lampu, semua berbau Art Deco. (Koran Tempo)Makna tersirat yang terdapat pada teks tersebut adalah . . . .a. setiap kota memiliki bangunan hotel kebanggan warganya sebagaimana warga Kota Bandung memiliki Hotel

Savoy Homann.b. Hotel savoy Homann menjadi kebanggaan warga Kota Bandung karena megah serta pernah ditempati

pemimpin serta seniman besar dunia.c. Bangunan hotel menjadi indah jika mengadopsi model bangunan Eropa sebagaimana model bangunan Hotel

Savoy Homann.d. Hotel menjadi kebanggaan warga jika memiliki sejarah panjang sebagaimana Hotel Savoy Homann yang

ditempati tokoh terkenal.e. Hotel Savoy Homann Bidakara yang berada di Kota Bandung bergaya Art Deco Eropa karena berbentuk

lengkung.69. Bacalah dengan cermat paragraf berikut!

NASA selesai uji coba sebuah sistem yang baru pertama kali diluncurkan, yaitu sebuah sistem “internet antarplanet”. Jaringan ini akan dapat dipergunakan secara otomatis untuk menyebarluaskan informasi ke bumi, wahana antariksa, dan para astronot. Wahana antariksa biasanya melakukan komunikasi langsung dengan bumi untuk memberikan instruksi kepada rover-rover Mars dalam misi eksplorasinya. Sprint dan opportunity mentransmisikan data ke orbiter lalu diteruskan kembali ke bumi. (Media indonesia, 231104). Gagasan utama paragraf tersebut adalah… a. NASA selesai menguji coba sistem komunikasi baru internet antarplanet. b. Jaringan internet antarplanet secara otomatis menyebarkan informasi. c. Jarngan internet antarplanet tidak memerlukan bantuan tenaga manusia. d. Wahana antariksa selalu melakukan komunikasi langsung dengan bumi. e. Sistem baru internet antarplanet NASA bernama sprint dan opportunity.

70. Bacalah dengan cermat paragraf berikut!Kanker paru saat ini sangat ditakuti sebagai salah satu penyakit yang mematikan. Hanya 25 % kanker jenis

ini bisa terdiagnosis pada tahap dini. Para ahli telah berupaya menemukan cara agar bisa mendeteksi sedini mungkin adanya kanker paru sehingga pasien bisa diselamatkan. Di antara mereka adalah peneliti dari University of Keltucky’sChandler Medical di Lexington, AS, yang mencoba teknik pemeriksaan antibodi dengan ketepatan tinggi untuk mendiagnosis sel kanker tahap dini. Mereka menganilisis 212 “calon” penderita kanker dengan semacam protein antibodi. Ternyata, protein tersebut sembilan puluh persen akurat. Cara ini bisa digunakan untuk membedakan antara tumor jinak dan kanker. Paragraf tersebut menjadi lengkap jika dipadukan dengan kalimat penutup sebagai berikut…. a. Karena itu, protein banyak dipakai untuk mendeteksi tumor jinak dan kanker. b. Dengan demikian, untuk mendeteksi kanker sedini mungkin, para ahli menggunakan protein antibodi. c. Oleh sebab itu, kanker pada saat ini tidak perlu ditakuti lagi karena telah ditemukan protein. d. Jadi, penderita kanker dapat mendeteksi secara dini dengan bantuan teknik antibodi. e. Akhirnya, untuk mendiagnosis sel kanker hanya dibutuhkan semacam protein lagi.

IV.TES POTENSI AKADEMIKA. Kemampuan Verbal

1. aliran: . . . = . . . : bersiha. air - udara d. mampat- kotoranb. darah- segar e. sungai- lingkunganc. saluran- bening

2. menghargai: . . . = … : kebenciana. hak milik – musuh d. toleransi – menghujatb. sesama- kekesalan e. menghormati- marahc. menjaga- menghina

3. penuh: . . .= . . . : berhentia. isi- gerakan d. meluap- melambatb. kosong- diam e. menggenang- mengeremc. banjir- pindah

4. remaja: . . .= . . .: pagia. jiwa- hari d. puber- waktub. usia- sinar e. penampilan- sarapan

Page 21: SBMPTN REVISI

c. dewasa- fajar5. mirip: . . .= . . .: laki- laki

a. wajah- peran d. tampilan- pemimpinb. serupa- gender e. berbeda- jenis kelaminc. kembar- maskulin

6. indah: . . . = . . .: tajama. seni- sakit d. taman- bahayab. lukisan- runcing e. kesenangan- lukac. perhiasan- pisau

7. tumbuh: … = . . . layua. hidup- mati d. puber- waktub. besar- lemah e. berkembang- meranggasc. subur- gugur

8. gembira: . . .= . . . : galaua. ria- berduka d. tertawa- kacaub. hati- merana e. bahagia- menangisc. histeria- gelisah

9. pengetahuan: . . .= . . .: biayaa. ilmu- ongkos d. cerdas- mahalb. bodoh- gratis e. sekolah- pasarc. dungu- harga

10. laut: . . .= . . .: kabupatena. pulau- peta d. nelayan- bupatib. ikan- daerah e. samudra- provinsic. air- wilayah

11. karet: . . .= . . .: populera. lentur- aktor d. ban- terkenalb. air- pendatang e. kenyal- tersohorc. kenyal- selebriti

12. …: tenteram= pengawasan: . . .a. aturan- ketat d. damai- pemeriksaanb. penjagaan- kontrol e. sejahtera- pengendalianc. keamanan- teratur

13. …:ceroboh= tepat: . . .a. lali- akurat d. hati- hati – melesetb. lengah- efektif e. waspada- tepat sasaranc. kecelakaan- efisien

14. ….: gila = lembab: . . .a. otak- kulit d. akal- keringb. aksi- basah e. saraf- ruangc. orang – udara

15. … : mencoblos= terbang: . . .a. bilik- angkasa d. memilih- mendaratb. gambar- pesawat e. kartu- layang- layangc. kandidat- burung

16. air: . . . = hampa: . . .a. sungai- ruang d. es- kosongb. kering- udara e. tetesan- vakum

c.dingin- pengap17. oven: . . .= . . . : foto

a. roti- kamera d. gosong- modelb. pemanggang- album e. matang- fotografer

c.panas- lensa18. murka: . . . = . . .: benturan

a. sedih- sakit d. cemberut- tendanganb. mencaci- luka e. kecewa- dorongan

c.marah- tabrakan19. buas: . . .= . . . : jet tempur

a. harimau- udara d. hiu- terbangb. serigala- manuver e. kecewa- dorongan

c. ular- laju20. hijau: . . . = . . . : laut

a. lumut- ikan d. daun- asinb. klorofil- ombak e. hutan- kapal

c.rumput- pantai21. gurun: . . . = . . .: hutan

Page 22: SBMPTN REVISI

a. pasir- belantara d. matahari- rimbunb. sahara- pohon e. kering- lebat

c.oasis- sungai22. produksi : . . . =. . . :perjalanan

a. pabrik- arah d. pekerja- kecepatanb. produk- tujuan e. upah- pulang

c.mesin- kendaraan23. buku: . . . = gawang: . . .

a. perpustakaan- lapangan bola d. ilmu- penjaga gawangb. bacaan- sepak bola e. rak- jaring

c.tulisan- pemain bola24. gerakan: . . . = kotoran: . . .

a. diam- sapu d. bergetar- berbaub. dinamis- limbah e. berpindah- pupuk

c.putaran- tercemar25. hari: … = titik: . . .

a. bulan- tanda baca d. minggu- garisb. tahun- koma e. september- kecil

c.kalender- menulis26. …: terang= suara: . . .

a. mata- bisu d. hitam- sunyib. warna- getaran e. cahaya- nyaring

c.kelam- ramai27. . …: petani= peluit: . . .

a. cangkul- wasit d. sawah- jurur parkirb. tanah- masinis e. bajak- sepak bola

c.padi- kereta api28. …: mobil= angin: . . .

a. kendaraan- udara d. jalan- baling- balingb. sedan- dingin e. rumah- butad. bensin- kincir

29. …: tuna wisma = penglihatana. harta- mata d. penghasilan- inderab. jalanan- kornea e. rumah- buta

c.spengangguran- katarak30. …: kelulusan= penghargaan: . . .

a. ujian- lomba d. ijazah- tanda jasab. siswa- pahlawan e. penilaian- hadiahc. tamat- medali

31. Gemetar berhubungan dengan . . ., sebagaimana . . . berhubungan dengan lucua. Lapar- geli d. takut- tertawab. Sakit- lelucon e. bahaya- pelawakc. Gugup- parodi

32. Merdeka berhubungan dengan . . ., sebagaimana . . . berhubungan dengan penjaraa. Pahlawan- narapidana d. bebas- jeruji besib. Proklamasi- sidang e. berontak- hubunganc. Penjajahan- kebebasan

33. Redup berhubungan dengan . . ., sebagaimana . . . berhubungan dengan teriaka. Cahaya- jauh d. seram- suarab. Terang- bisik e. lampu- memanggilc. Gelap- terkejut

34. Senja berhubungan dengan . . ., sebagaimana . . . berhubungan dengan berlaria. Malam- berjalan d. siang- kejarb. Petang- kejar e. bulan- berdiric. Waktu- kejar

35. Matematika berhubungan dengan . . ., sebagaimana . . . berhubungan dengan mobila. Matriks- transportasi d. angka- kendaraanb. Rumus- jalan e. hitung- mesinc. Pelajaran- sedan

36. Laptop berhubungan dengan . . ., sebagimana . . . dengan busanaa. Komputer- kaos d. layar- kaosb. Printer- sandal e. internet- semirc. Telepon genggam- tali

37. Ulat berhubungan dengan . . ., sebagaimana . . . berhubungan dengan berudu (kecebong)a. Bulu- kolam d. pohon- enceng gondokb. Larva- ikan e. kepompong- katak

Page 23: SBMPTN REVISI

c. Kupu- kupu- katak hijau38. Wisuda berhubungan dengan . . ., sebagaimana . . . berhubungan dengan pertunangan

a. Toga- cincin d. sarjana- mempelaib. Gelar- pelaminan e. kuliah- pernikahanc. Berhasil- cinta

39. Sarung tangan berhubungan dengan . . ., sebagaimana . . . berhubungan dengan bakteriologa. Steril- stetoskop d. kotor- kumuhb. Ahli forensik- mikroskop e. bersih- bakteric. Dokter- apoteker

40. Benda bersejarah berhubungan dengan . . ., sebagaimana . . . berhubungan dengan kebun binatanga. Museum- margasatwa d. peninggalan- keanekaragamanb. Galeri- taman wisata e. kenangan- keindahanc. Patung- burung

41. Menguap berhubungan dengan . . ., sebagaimana . . . berhubungan dengan sakita. Istirahat- panas badan d. tidur- istirahatb. Lelah- dokter e. tempat tidur- obatc. Mengantuk- demam

42. Kaku berhubungan dengan . . ., sebagaimana . . . berhubungan dengan kareta. Tongkat-gelang d. kaki- fleksibelb. Batu- lembut e. kayu- lateksc. Besi- lentur

43. Sakit berhubungan dengan . . ., sebagaimana . . . berhubungan dengan makana. Dokter- sakit d. berobat- gemukb. Makan- lemah e. pasien- gizic. Istirahat- makan

44. Marah berhubungan dengan . . ., sebagaimana . . . berhubungan dengan ledakana. Kerusakan- peledak d. tidak puas- terorb. Amuk- letupan e. hawa nafsu- panasc. Emosi- bom

45. Lapangan bola berhubungan dengan . . ., sebagaimana . . . berhubungan dengan dekorasia. Tribun- bunga d. wasit- tukang fotob. Pemain bola- seniman e. tiang gawang- panggungc. Bersepatu- bercahaya

46. Bayi berhubungan dengan . . ., sebagaimana . . . berhubungan dengan dewasaa. Berjalan- anak d. elajar- mudab. Lemah- kuat e. merangkak- pemudac. Anak- remaja

47. Vonis berhubungan dengan . . ., sebagaimana . . . berhubungan dengan pertandingana. Tuntutan- juara d. penjara- pialab. Peradilan- hasil e. hakim- suporterc. Pengacara- wasit

48. Toko berhubungan dengan . . ., sebagaimana . . . berhubungan dengan ikana. Kasir- akuarium d. makanan- pancingb. Pembeli- cacing e. pakaian- bakarc. Supermarket- paus

49. Bunyi alarm berhubungan dengan . . ., sebagaimana . . . berhubungan dengan apia. Sinyal- air d. kebakaran- arangb. Peringatan- padam e. bahaya- asapc. Suara- abu

50. Peluit berhubungan dengan . . ., sebagaimana . . . berhubungan dengan prajurita. Wasit- senjata d. permainan- perangb. Polisi –markas e. kereta- pasukanc. Penalti- desersi

51. Nelayan : lauta. Guru : papan tulis d. pelukis: kuasb. Petani: padi e. penyanyi: panggungc. Karyawan: pegawai

52. Samudra: lauta. Benua: pulau d. negara: pantaib. Internasional: batas negara e. luas: dalamc. Kapal: nelayan

53. Kuda laut: kudaa. Kucing: harimau d. cacing: belutb. Banteng: sapi e. merpati: elangc. Biawak: komodo

54. Beranak: mamalia

Page 24: SBMPTN REVISI

a. Bertaring: karnivora d. berjemur: reptiliab. Bertelur: unggas e. berkaki empat: herbivorac. Berkelompok: insekta

55. Kurus: gizia. Gemuk: lemak d. pintar: belajarb. Sakit: dokter e. bodoh: ilmuc. Pendek: besar

56. Gigi: kunyah = jari: …a. Lentik b. pegang c. kuku d. tangan e. manis

57. Melati: bunga= bawang: …a. Umbi b. merah c. putih d. bumbu e. harum

58. … berhubungan dengan padi, sebagaimana buah berhubungan dengan . . .a. Kuning- merah d. sawah- vitaminb. Makanan pokok- minuman pokok e. beras- bijic. Petani- pedagang

59. … berhubungan dengan buruh, sebagaimana supervisor berhubungan dengan . . .a. Mandor- karyawan d. demo- kerjab. Pabrik- kantor e. upah- gajic. Kecil- besar

60. … berhubungan dengan puisi, sebagaimana pemahat berhubungan dengan . . .a. Chairil Anwar- seniman d. penyair- pematungb. Syair- batu e. suara- gerakc. Baca- seni

61. Gelap= . . .a. Kelam b. kilau c. silau d. muram e. gerah

62. Pakar= . . .a. Ahli b. kompleks c. umum d.khusus e. golongan

63. Adaptasi= . . .a. tetap b. penyesuaian c. pilihan d. perencanaan e. perubahan

64. Subsantsi= . . .a. nyata b. maka c. maksud d. inti e. isi

65. Resah= . . .a. Gulana b. lekat c. gundah d. kendala e. dilema

66. Stabil >< …a. Buruk b. insidentil c. kuat d. permanen e. labil

67. Parsial >< . . .a. Internal b. eksternal c. komunal d. komuniter e. konflik

68. Permanen >< . . .a. Kuat b. sementara c. komunal d. induk e. manual

69. Versus >< . . .a. Lawan b. mitra c. pengaruh d. cegah e. ancam

70. Parasit >< . . .a. Konfusit b. inplisit c. trombosit d. analis e. simbolis

71. Gudang : baranga. Lama: baru d. bank: depositb. Buku: perpustakaan e. bangunan- antikc. Tertutup- terbuka

72. Buruh: pabrika. Penari: studio d. pemahat: candib. Pedagang: pasar e. petani: sungaic. Nelayan: pesisir

73. Kering: lembaba. Gelap- remang- remang d. matahari- gunungb. Cahaya- terang e. siang- malamc. Panas- basah

74. Paus: hiua. Baiwak: komodo d. singa: harimaub. Belut: cacing e. kambing: sapic. Merpati: elang

75. Memberi: dermawana. Mencuri: polisi d. mencuci: bajub. Melukis: kanvas e. menulis: sekretarisc. Membantu- orang

76. Kehausan: aira. Kemakmuran- kerja keras d. kelelahan- kerjab. Kebodohan- miskin e. kelaparan- pangan

Page 25: SBMPTN REVISI

c. Kegembiraan- hiburan77. Terbang berhubungan dengan . . ., sebagaimana jalan berhubungan dengan . . .

a. Pesawat- jauh d. burung- kupu- kupub. Burung- unta e. pesawat- daratc. Tinggi- jauh

78. Ayah berhubungan dengan . . ., sebagaimana pohon berhubungan dengan ….a. Orang tua- daun d. anak- akarb. Manusia- tumbuhan e. anak- tunasc. Laki- laki- ranting

79. Es berhubungan dengan . . ., sebagaimana gula berhubungan dengan . . . .a. Padat- bubuk d. dingin- manisb. Cair- kristal e. beku- kristalc. Cair- padat

80. Mobil berhubungan dengan . . ., sebagaimana kompor berhubungan dengan . . . .a. Bahan bakar- laut d. solar- kayub. Kendaraan- dapur e. busi-sumbuc. Bensin- gas

81. Pergi berhubungan dengan . . ., sebagaimana begadang berhubungan dengan. . .a. Jauh- malam d. pulang- malamb. Keluar- ronda e. jauh- tidurc. Pulang- tidur

82. …: komputer = . . .: manusiaa. Mouse- tangan d. CPU- tubuhb. Monitor- mata e. keyboard- jaric. Prosesor- otak

83. . . .: mobil= . . .: sampana. Bensin- dayung d. darat- airb. Kendaraan- perahu e. kendaraan- angkutanc. Darat- laut

84. . . .: pergi= . . .: awala. Datang- akhir d. pulang- zamanb. Pulang- belakang e. awal- mukac. Berangkat- kedatangan

85. . . .: makan= . . .: obata. Nasi- resep d. nasi- lukab. Lapar- minum e. restoran- apotekc. Lapar- sakit

B. Pemahaman Wacana Teks I

Toni memiliki sebuah rak buku kecil yang terbagi atas lima tingkat, tingkat 1,2,3,4, dan 5. Tingkat I adalah yang teratas dan tingkat 5 adalah yang terbawah. Toni menyimpan buku-buku pelajaran ke dalam rak dengan aturan sebagai berikut:

a. Buku pelajaran geografi disimpan tepat di atas buku pelajaran matematika, sedangkan buku pelajaran bahasa Indonesia diletakkan tepat di bawah buku matematika.

b. Buku pelajaran ekonomi diletakkan di atas buku pelajaran fisika, sedangkan buku pelajaran fisika dan kimia berada di atas buku pelajaran geografi.

c. Buku pelajaran kimia dan biologi diletakkan saling terpisah sejauh mungkin.Tabel untuk jawaban nomor 1- 6

ekonomi, kimiafisika geografimatematikabahasa Indonesia, biologi

1. Buku yang disimpan tidak berdampingan dengan buku yang lain adalah . . . .a. Fisika, matematika, geografib. Kimia, fisika, biologic. Matematika, bahasa Indonesia, ekonomid. Bahasa Indonesia, biologi, geografie. Ekonomi, kimia, fisika

2. Adik Toni mengambil buku geografi dan menyimpannya kembali di samping buku bahasa Indonesia. tingkatan yang kosong adalah tingkatan ke . . . .a. 1 b. 2 c.3 d.4 e. 5

3. Buku yang terletak di tingkat 1 adalah buku . . . .a. Biologi b. fisika c. geografi d. kimia e. matematika

Page 26: SBMPTN REVISI

4. Buku yang letaknya setingkat dengan buku bahasa Indonesia adalah buku . . . .a. Biologi b. ekonomi c. fisika d. geografi e. matematika

5. Buku yang disimpan di tingkat ketiga adalah buku . . . .a. Biologi b. ekonomi c. fisika d. geografi e. matematika

6. Urutan lima buku dari tingkat tertinggi ke tingkat terendah adalah . . . .a. Kimia, fisika, geografi, matematika, biologib. Bahasa Indonesia, biologi, geografi, kimiac. Ekonomi, fisika, matematika, geografi, biologid. Fisika, matematika, geografi, bahasa Indonesiae. Matematika, geografi, bahasa Indonesia, biologi, fisika

Teks IIA,B,C,D, dan E adalah lima pekerja yang mencalonkan diri menjadi ketua serikat kerja. Ada tiga program yang diajukan para calon, yaitu (1) program peningkatan keterampilan pekerja oleh A,B, dan C. (2) program olahraga oleh B,C, dan D (3) program peningkatan kesejahteraan oleh D,E, dan A. Setiap warga hanya diperkenankan memilih satu calon berdasarkan progamnya. Hasil pemungutan suara adalah sebagai berikut:

a. Enam puluh pekerja memilih program peningkatan keterampilan pekerjab. Empat puluh lima pekerja memilih program olahragac. Sembilan puluh pekerja memilih program peningkatan kesejahteraand. Jumlah suara terbagi sama besar pada calon- calon yang memiliki program sama

Tabel untuk jawaban nomor 7- 111) Program peningkatan keterampilan pekerja2) Program olahraga3) Program peningkatan kesejahteraan

Calon 1 2 3 Jumlah SuaraA 20 - 30 50B 20 15 - 35C 20 15 - 35D - 15 30 45E - - 30 30

Jumlah 60 45 907. Calon yang memperoleh suara terbanyak adalah . . . .

a. A b. B c. C d. D e. E8. Calon yang memperoleh suara paling sedikit adalah . . .

a. A b B c. C d. D e. E9. A dan B memilik perbedaan perolehan suara sebesar . . . .

a. 5 b. 10 c. 15 d. 20 e. 2510. Urutan dengan perolehan tiga suara terbanyak, dinulai dari yang terbanyak adalah . . . .

a. ADB b. ABC c.CAB d. DBE e. ECA11. Jika ketua terpilih mengangkat ketua seksi olahraga dari calon yang memperoleh suara terbanyak dan mengajukan

program olahraga, yang menjadi ketua seksi olahraga adalah . . . /a. A b. B c. C d. D e. E

Teks IIITono akan ke luar negeri selama satu bulan untuk urusan usaha dagangnya sehingga ia harus memilih salah satu dari tiga bawahannya yang akan menggantikannya di kantor. Beberapa informasi yang menjadi dasar keputusan Tono sebagai berikut:

a. Anto cukup kreatif dan cukup baik memimpin, namun sering sakit sehingga terkesan kurang rajin bekerja.b. Hasan paling kreatif dan cukup sehat, tetapi masih kalah dari Rudi dalam hal kerajinan dan masih kalah

dari Anto dalam kepemimpinan.c. Dalam kreativitas, Rudi masih di bawah Hasan dan Anto, tetapi ia paling rajin, paling sehat, dan paling

bagus memimpin.12. Jika Tono mempertimbangkan aspek kesehatan dan kerajinan sebagai kriteria utama dalam memilih wakilnya,

pernyataan yang paling tepat adalah . . .a. Anto berpeluang besar. d. Anto dan Rudi berpeluang sama besar.b. Hasan berpeluang besar. e. Anto, Hasan, Rudi berpeluang sama.c. Rudi berpeluang paling besar.

13. Pernyataan yang tidak tepat adalah . . . .a. Anto dan Hasan masih kalah sehat dibandingkan Rudi.b. Anto dan Rudi mengalahkan Hasan dalam kepemimpinan.c. Kekuarangan Anto dibandingkan Hasan dalam hal kreativitas.d. Kekurangan Anto dibandingkan Hasan dan Rudi adalah dalam hal kerajinan dan kesehatan.e. Anto lebih rajin daripada hasan dan lebih abik dalam memimpin.

14. Berdasarkan keunggulan – keunggulan aspeknya, calon pengganti Tono yang berpeluang paling besar secara berurutan adalah . . . .a. Anton, Rudi, Hasan d. Anton, Hasan, Rudib. Rudi, Anton, Hasan e. Rudi, Hasan, Anto

Page 27: SBMPTN REVISI

c. Hasan, Rudi, Anto15. Jika Tono mempertimbangkan aspek kreativitas dan kepemimpinan sebagaikriteria utama, pernyataan yang paling

tepat adalah . . . .a. Anto berpeluang terbesar mewakilinyab. Hasan berpeluang terbesar mewakilinyac. Rudi berpeluang terbesar mewakilinyad. Anto dan Rudi berpeluang sama besar mewakilinyae. Anto, Hasan, san Rudi berpeluang sama besar mewakilinya

Teks IVSembilan murid pria dan wanita A,B,C,D,E,F,G,H dan I secara bersama membentuk barisan. Baris depan diisi oleh A,B, dan C. Baris tengah diisi oleh D,E, dan F, sedangkan baris belakangdiisi oleh G, H, dan I. Sisi kiri barisan diisi oleh B,F, dan I,sedangkan sisi kanan diisi oleh A,E, dan H. Kecuali pada barisan- barisan tersebut bercirikan sebagai berikut:

a. Murid pria dan wanita berselang- selingb. Pada barisan yang beranggotakan A, barisan terdiri atas dua muria wanita dan satu murid pria.

16. Setiap barisan diagonal terdiri atas . . . .a. Dua wanita, satu pria d. tiga priab. Dua pria, satu wanita e. tiga wanitac. Tiga pria atau tiga wanita

17. Murid-murid yang ada di tengah barisan urut dari depan ke belakang adalah . . . a. C,G,D b.C,D,G c.D,C,G d. E,D,F e. F,D,E

18. Berapakah murid wanita di antara sembilan murid tersebut?a. 2 b. 3 c. 4 d. 5 e. 6

19. Jenis kelamin B, E, dan I adalah . . . .a. B wanita, E pria, I wanita d. B wanita, I pria, E wanitab. E wanita, B pria, I wanita e.E wanita, B wanita, I priac. I wanita, B pria, E pria

Teks VDalam pertandingan bulu tangkis Arman selalu kalah melawan Bambang, tetapi dalam cabang olahraga yang lainnya ia selalu menang bila melawan Bambang. Candra selalu menang dalam pertandingan tenis meja melawan Bambang, tetapi dalam cabang bulu tangkis ia akan kalah bila bertanding melawan Arman. Dudi adalah pemain bulu tangkis terbaik, tetapi dalam cabang tenis meja dia tidak sebaik Bambang. Dalam cabang tenis meja, Edi lebih baik daripada Arman, sedangkan dalam cabang bulu tangkis ia menempati urutan tepat di bawah Dudi.

20. Siapakah pemain tenis meja terbaik di antara kelima atlet tersebut?a. Arman b. Bambang c. Candra d. Dudi e. Edi

21. Untuk cabang olahraga tenis meja, ranking pertama terbaik yang manakah yang paling tepat dari urutan di bawah ini?a. Bambang- Arman- Candra- Dudi- Edi d. Edi- Dudi- Candra- Bambang- Armanb. Arman- Bambang- Candra- Edi- Dudi e. Edi- Arman- Candra- Bambang- Dudi c. Dudi- Edi- Candra- Bambang- Arman

22. Untuk cabang olahraga bulu tangkis, ranking pemain terbaik manakah yang paling tepat dari urutan di bawah ini?a. Dudi- Edi- Arman- Bambang- Candra d. Bambang- Dudi- Edi- Arman- Candrab. Bambang- Arman- Dudi- Edi- Candra e. Dudi- Edi- Candra- Bambang- Armanc. Dudi- Edi- bambang- Arman- candra

Teks VIMinggu ini yang mendapat giliran piket kelas adalah tiga anak wanita, yaitu Ita, Ayu, dan Lani, dan dua anak pria, yaitu Putra dan Dani. Setiap hari harus ada tiga orang yang membersihkan kelas mulai dari Senin sampai Jumat dengan ju,lah yang sama. Pengaturan jadwal harus mempertimbangkan hal-hal berikut:

a. Setiap Jumat Ita dan Dani ikut kegiatan pramuka sehingga tidak dapat membersihkan kelas.b. Setiap Senin dan Rabu Ayu harus segera pulang untuk menjemput adiknya di TK.c. Setiap hari harus ada anak pria yang membersihkan kelas.d. Lani harus segera pergi ke tempat les matematika setiap Senin dan Kamis.

23. Siswa yang membersihkan kelas pada tiap Selasa adalah . . . .a. Dani, lani, dan Ita d. Lani, Ayu, dan Danib. Putra, Ita, dan Dani e. Ita, Dani, dan Putrac. Lani, Ita, dan Putra

24. Ita dan Dani mendapat giliran bekerja membersihkan kelas setiap . . . .a. Selasa dan Kamis d. rabu dan Jumatb. Senin dan Selasa e. Rabu dan Kamisc. Senin dan Kamis

25. Dani dan Putra tidak pernah mendapat giliran meambersihkan bersama- sama, kecuali pada . . . .a. Senin b. Selasa c. Rabu d. Kamis e. Jumat

Teks VIISetiap berangkat ke sekolah Nina membawa bekal makanan. Bekal makanan yang dibawa Nina diatur oleh ibunya tanpa memperhitungkan hari ketika Nina tidak masuk sekolah. Nasi goreng dan nasi putih dibawa Nina secara bergantian setiap sekolah. Lauk untuk dua hari sekolah berurutan berupa ayam goreng baru kemudian telur

Page 28: SBMPTN REVISI

goreng untuk satu hari sekolah berikutnya, dan seterusnya berulang.a. Hari pertama pada minggu pertama tahun ajaran adalah Senin.b. Bekal pertama kali yang dibawa Nina berupa nasi goreng dan ayam goreng.c. Nina tidak masuk sekolah pada Selasa dan Sabtu minggu kedua karena sakit.d. Rabu minggu kelima adalah hari libur besar nasional.

Tabel jadwal membawa bekal (tabel yang sama untuk empat minggu berikutnya)Senin nasi goreng dan ayam gorengSelasa nasi putih dan ayam gorengRabu nasi goreng dan telur gorengKamis nasi putih dan ayam gorengJumat nasi goreng dan ayam gorengSabtu nasi putih dan telur gorengMinggu nasi goreng dan ayam goreng

26. Pada hari apakah, selama lima minggu pertama Nina dibekali dengan nasi goreng dan ayam goreng?a. Senin b. Selasa c. Rabu d. Kamis e. Jumat

27. Pada hari apakah selama lima minggu pertama Nina lebih sering membawa bekal nasi putih dan telur goreng?a. Senin b. Selasa c. Rabu d. Kamis e.Jumat

28. Pada hari apa sajakah, pada minggu ketiga, Nina dibekali nasi putih dan ayam goreng?a. Senin dan Rabu d. Kamis dan Sabtub. Selasa dan Kamis e. Jumat dan Seninc. Rabu dan Jumat

29. Selam lima minggu pertama berpa kalikah Nina membawa bekal nasi goreng dan telur goreng?a. 8 b. 7 c. 6 d. 5 e. 4

30. Bacalah dengan cermat teks berikut!Seorang karyawan mengatur enam ruang kerja untuk enam staf dengan urutan ruang nomor 1 sampai 6 dengan aturan sebagai berikut!1. Bu Rati sering bercakap- cakap yang suaranya terdengar keras ke ruang sebelahnya.2. Pak Mara dan Pak Bono ingin berdekatan agar dapat berkoordinasi. 3. Bu Heni meminta ruang nomor 5 yang berjendela lebar.4. Pak Dedi tidak suka pekerjaannya terganggu oleh suara- suara. 5. Pak Tasman, Pak mara, dan pak Dedi adalah perokok.6. Bu Heni alergi dengan asap rokok.Dari keterangan yang diperoleh, pengaturan ruangan yang tepat adalah sebagai berikut:Dedi Tasman Mara Bono Heni RatiTiga karyawan perokok seharusnya ditempatkan di ruang nomor . . . .a. 1,2, dan 4 d. 2,3, dan 4b. 2,3, dan 6 e. 1,2, dan 6c. 1,2, dan 3

71. Curik bali yang lebih dikenal dengan jalak bali beberapa tahun lagi mungkin sudah hilang habitatnya di Taman nasional Bali Barat. Taman nasional satu-satunya di Bali ini merupakan satu-satunya “rumah” bagi curik bali. Sayangnya, kini taman nasional bukan lagi rumah yang aman dan nyaman bagi burung yang anggun itu. Topik paragraf tersebut adalah….a. Hilangnya curik bali dari habitatnya. b. Tidak aman dan nyamannya curik bali di habitatnya. c. Ditemukannya curik bali di Taman Nasional Bali Barat. d. Satu-satunya taman nasional di bali sebagai rumah dari curik bali.

Page 29: SBMPTN REVISI

e. Tidak aman dan nayamannya bagi curik bali. 72. NASA selesai uji coba sebuah sistem yang baru pertama kali diluncurkan, yaitu sebuah sistem “internet

antarplanet”. Jaringan ini akan dapat dipergunakan secara otomatis untuk menyebarluaskan informasi ke bumi, wahana antariksa, dan para astronot. Wahana antariksa biasanya melakukan komunikasi langsung dengan bumi untuk memberikan instruksi kepada rover-rover Mars dalam misi eksplorasinya. Sprint dan opportunity mentransmisikan data ke orbiter lalu diteruskan kembali ke bumi. (Media indonesia, 231104). Gagasan utama paragraf tersebut adalah… f. NASA selesai menguji coba sistem komunikasi baru internet antarplanet. g. Jaringan internet antarplanet secara otomatis menyebarkan informasi. h. Jarngan internet antarplanet tidak memerlukan bantuan tenaga manusia. i. Wahana antariksa selalu melakukan komunikasi langsung dengan bumi. j. Sistem baru internet antarplanet NASA bernama sprint dan opportunity.

73. Saat ini diperlukan realisasi janji dari para pemimpin negeri ini, yakni peningkatan kualitas pendidikan dengan anggaran hingga 20% dari total APBN dan APBD. Sayangnya, baru sekitar 44 kabupaten di Indonesia yang mengalokasikan anggaran pendidikan sesuai dengan ketentuan tersebut. Padahal, peningkatan kualitas pendidikan ini telah masuk dalam delapan prioritas pembangunan dalam program Susilo Bambang Yudhoyono. Dalam salah satu dari prioritas itu disebutkan dengan jelas bahwa peningkatan akses dan kualitas pendidikan dan kesehatan menjadi prioritas pembangunan indonesia. Namun, realisasinya masih sangat mengecewakan. (Koran Sindo, 211108). Pernyataan berikut yang tidak relevan dengan isi teks tersebut adalah…. f. Pemerintah menjanjikan anggaran pendidikan sebesar 20% dari APBN dan APBD. g. Alokasi anggaran pendidikan 20% baru dilakukan oleh 44 kabupaten. h. Janji pemerintah menetapkan anggaran pendidikan 20% belum ditepati. i. Pendidikan dan kesehatan merupakan prioritas utama program. j. Realisasi dari delapan prioritas pembangunan sangat mengecewakan.

74. …. Salah satu penyakit kulit adalah acne vulgaris. Tidak ada seorang pun di dunia ini yang tidak pernah menderita penyakit ini. Acne vulgaris adalah penyakit peradangan folikel sebasea yang umumnya terjadi pada masa remaja dan dapat sembuh sendiri. Dalam masyarakat umum acne vulgaris biasa dikenal dengan istilah jerawat. Kalimat yang manakah yang tepat untuk mengisi bagian rumpang tersebut adalah… f. Acne vulgaris adalah penyakit kulit biasa. g. Acne vulgaris pernah dialami semua manusia. h. Penyakit kulit banyak jenis dan macamnya. i. Jerawat banyak jenis dan macamnya. j. Penyakit yang diderita remaja bermacam-macam.

75. Pasar uang dan pasar modal Indonesia belum pulih. Rupiah masih terus mengalami tekanan mengikuti penurunan sebagian mata uang Asia. Masalah minimnya pasokan dolar AS di pasar valas semakin mempersulit mata uang lokal ini. Pada penutupan perdangan valas tanggal 24 November 2008, rupiah melemah hingga 320 poin ke posisi 12. 320 per dolar AS. Rupiah bahkan sempat menembus 12.325 per dolar AS. Pelaku pasar cemas karena belum ada sentimen positif dari dalam negeri. Oleh karena itu, pasar lebih memilih memegang dolar AS dalam kondisi pasar global yang sedang rentan ini. Paragraf tersebut dikembangkan dengan menggunakan pola… b. sebab-akibat b. akibat-sebab c. generalisai d. analogi e. silogisme

76. Kanker paru saat ini sangat ditakuti sebagai salah satu penyakit yang mematikan. Hanya 25 % kanker jenis ini bisa terdiagnosis pada tahap dini. Para ahli telah berupaya menemukan cara agar bisa mendeteksi sedini mungkin adanya kanker paru sehingga pasien bisa diselamatkan. Di antara mereka adalah peneliti dari University of Keltucky’sChandler Medical di Lexington, AS, yang mencoba teknik pemeriksaan antibodi dengan ketepatan tinggi untuk mendiagnosis sel kanker tahap dini. Mereka menganilisis 212 “calon” penderita kanker dengan semacam protein antibodi. Ternyata, protein tersebut sembilan puluh persen akurat. Cara ini bisa digunakan untuk membedakan antara tumor jinak dan kanker. Paragraf tersebut menjadi lengkap jika dipadukan dengan kalimat penutup sebagai berikut…. f. Karena itu, protein banyak dipakai untuk mendeteksi tumor jinak dan kanker. g. Dengan demikian, untuk mendeteksi kanker sedini mungkin, para ahli menggunakan protein antibodi. h. Oleh sebab itu, kanker pada saat ini tidak perlu ditakuti lagi karena telah ditemukan protein. i. Jadi, penderita kanker dapat mendeteksi secara dini dengan bantuan teknik antibodi. j. Akhirnya, untuk mendiagnosis sel kanker hanya dibutuhkan semacam protein lagi.

77. Di dunia pendidikan, penilaian secara garis besar terbagi menjadi dua, yaitu penilaian internal dan penilain eksternal. Penilaian internal dilakukan untuk mengetahui seberapa efektif kegiatan pembelajaran yang dilakukan oleh guru. Hasilnya diharapkan dapat memperbaiki institusi penyelenggara. Penilaian eksternal yang berfungsi sebagi penekan ini perlu dilakukan karena biasanya justru menjadi alat efektif untuk mendorong sekolah tersebut bergerak ke arah perbaikan. Demi melengkapi informasi, kalimat “Tujuannya adalah untuk mendapatkan umpan balik sekaligus memantau kemajuan belajar anak.” Dapat ditambahkan dalam paragraf tersebut. Letak yang paling tepat adalah setelah kalimat …. a. pertama b. kedua c. ketiga d. keempat e. kelima

78. Selain kebakaran hutan, ancaman paling serius terhadap hutan di Indonesia adalah pembalakan secara liar. Hutan Indonesia terancam musnah. Dari sudut ekologi, kerusakan hutan yang parah menimbulkan dampak negatif

Page 30: SBMPTN REVISI

terhadap lingkungan yang harus ditanggung dalam jangka panjang. Dari sisi ekonomi, dua tahun terakhir saja, negara dirugikan lebih dari Rp90 triliun. Pernyataan berikut tidak sesuai dengan isi teks tersebut, kecuali… a. Pembalakan secara liar berbahaya daripada kebakaran hutan. b. Pembalakan secara liar artinya penebangan pohon di hutan tanpa izin formal. c. Perekonomian negara akan memburuk jika pembalakan secara liar terus terjadi. d. Ekologi adalah suatu cabang ilmu yang mempelajari masalah lingkungan hidup. e. Penyebab utama kerusakan hutan di Indonesia adalah pembalakan secara liar.

79. Daging merupakan bahan pangan hewani yang digemari seluruh lapisan masyarakat karena rasanya yang lezat dan bergizi. Bila dibandingkan dengan bahan pangan nabati, daging merupakan sumber protein yang lebih baik karena mengandung asam-asam amino esensial yang lebih lengkap dan seimbang serta lebih mudah dicerna. Selain itu, daging merupakan sumber lemak, vitamin, dan mineral. Setiap seratus gram daging rata-rata dapat memenuhi sebesar 10% kalori, 50% protein, 35% zat besi (Fe), dan 25%-60% vitamin B kompleks dari kebutuhan gizi orang per hari. Yang dipaparkan dalam paragraf tersebut adalah….a. Pentingnya bahan pangan dari daging bagi manusia. b. Alasan bahan pangan dari daging digemari orang. c. Uraian tentang daging sebagai bahan pangan hewan. d. Perbandingan daging dengan bahan nabati.e. Kandungan gizi pada daging.

80. Berdasarkan hasil penelitian, perkembangan penggunaan varietas padi ketan unggulDi Indonesia relatif lambat. Hal ini ditunjukkan oleh rendahnya angka serapan pemanfaatan benih padi ketan varietas Lusi, Ketonggo, dan Setail. Belum banyak petani yang mengetahui ketiga varietas padi ketan ini, apalagi menanamnya. Kondisi tidak lepas dari lambatnya penyampaian informasi ketersediaan varietas tersebut kepada petani. Selain itu, iklim usaha beras ketan tampaknya juga belum mendukung. Paragraf tersebut dikembangkan dari kalimat inti… a. Perkembangan penggunaan varietas padi ketan unggul lambat. b. Banyak petani yang belum mengetahui varietas padi ketan. c. Lambatnya penyampaian informasi dan iklim usaha beras ketan belum mendukung. d. Rendahnya angka serapan pemanfaatan penggunaan varietas padi ketan unggul. e. Hasil penelitian menunjukkan lambatnya perkembangan penggunaan varietas padi ketan unggul.

81. Belakangan ini, virus lokal merajalela. Berbagai macam pola serangan dan cara kerja virus menandakan bahwa pembuat virus di indonesia semakin kreatif meski sering merepotkan. Varian terbaru dari virus lokal adalah virus loren. Cara kerjanya mengacaukan sistem windows dan memberikan kata-kata indah setiap kali kita login ke windows. Sebenarnya virus ini tidak berbahaya, hanya menduplikat folder yang terdapat di setiap partisi hard disk dengan file aplikasi dan nama yang menyerupai nama folder sehingga membuat pengguna kebingungan. Pernyataan berikut yang sesuai dengan isi paragraf tersebut adalah….a. Pembuat virus loren adalah orang Indonesia. b. Virus loren merupakan salah satu dari virus-virus lokal yang dapat memberikan kata-kata indah. c. Pengguna harus ekstra bersabar karena adanya sistem duplikasi dalam sistem windowsnya. d. Virus lokal ini dianggap kreatif karena dapat mengacaukan sistem dan memunculkan kata-kata indah. e. Virus loren dapat menyerang sistem windows, namun tidak berbahaya.

82. Dalam masyarakat Jawa, upacara selamatan hingg kini masih menjadi aktivitas sosial. Hampir seluruh kalangan masyarakat Jawa jika memunyai hajatan apa pun, memandang perlu melakukan upacara selamatan. Upacara selamatan dipandang sebagai sebuah aktivitas yang sangat penting sebagai upaya untuk mencari keselamatan serta ketenteraman sekaligus menjaga kelestarian kosmos. Aktivitas selamatan merupakan salah satu upacara upaya ritual sebagai jembatan antara dunia bawah, yaitu manusia, dengan dunia atas, yaitu makhluk halus atau Tuhan. Gagasan utama paragraf terebut adalah… a. Pentingnya uacara selamatan bagi masyarakat Jawa. b. Upacara selamatan sebagai upacara tradisional masyarakat Jawa. c. Upacara selamatan sebagai cara mencari keselamatan dan ketenteraman di kalangan masyarakat Jawa. d. Upacara selamatan sebagai satu-satunya upacara ritual di masyarakat Jawa. e. Upacara selamatan sebagai aktivitas sosial.

83. Beberapa tahun terakhir, animo mahasiswa asing menuntut ilmu di kampus-kampus Indonesia merambat naik. Mereka tersebar di berbagai universitas, baik negeri maupun swasta di seluruh Indonesia. Ilmu yang dipelajari pun beragam, mulai dari ilmu sosial hingga eksata. Ada yang mengambil kelas internasional dengan pengantar bahasa Inggris, ada pula kelas reguler dengan pengantar bahasa Indonesia. Paragraf tersebut membicarakan…. a. Maraknya kegiatan mahasiswa asing di Indonesia. b. Perkembangan minat mahasiswa asing untuk belajar di Indonesia. c. Persebaran mahasiswa asing yang belajar di universitas-universitas di Indonesia. d. Aneka bidang studi yangdapat dipelajari mahasiswa asing di Indonesia. e. Kelas internasional dan reguler yang dapat diikuti mahasiswa asing di Indonesia.

84. Empat desa dari delapan desa terendam lumpur panas di kabupaten Sidoarjo, Jawa Timur. Keempat desa itu, yakni Jatirejo, Siring, Kedungbendo, dan Renokenongo, tidak mungkin lagi ditempati karena ketinggian lumpur sudah

Page 31: SBMPTN REVISI

lebih dari empat meter dan menenggelamkan rumah penduduk serta infrastruktur lainnya. Selain itu, pemukiman tanah di empat desa terebut sudah turun 23-88 sentimeter hanya dalam waktu satu bulan terakhir. Paragraf tersebut akan menjadi paragraf padu (koheren) jika dilengkapi kalimat penjelas berikut….a. Demikian pula, lumpur panas yang menyebar menjadi ancaman bagi warga. b. Karena itu, kalaupun semburan lumpur dihentikan, daerah tersebut terlalu berbahaya. c. Sementara keempat desa yang lain dinilai tidak layak dihuni. d. Genangan lumpur di pinggiran tanggul-tanggul kini makin tinggi. e. Warga yang tidak mau pindah ke tempat yang disediakan akan diganti rugi dalam batas kewajaran.

85. Sejumlah pengusaha sepatu kecil-menengah di Kabupaten Mojokerto mengalami kesulitan menghadapi aturan perdagangan bebas atau free trade agreement (FTA) ASEAN- China. Mereka mengaku merugi sejak isu itu digulirkan pada awal Januari lalu. Para pedagang mengalami dampak negatif aturan itu. pada sektor usaha kecil sepatu, misalnya pengusaha merugi sampai 50%. “Orderan berkurang 50%. Artinya, produksi kami juga berkurang sebesar itu,” kata Budi Utomo, anggota Gabungan Pengusaha Sepatu (GPS) Kabupaten Mojokerto. Sepinya order terjadi karena sebagian besar pelanggan tidak melakukan pemesan- an karena menunggu masuknya prodruk impor lebih murah daripada harga produk lokal. Pernyataan berikut yang tidak sesuai dengan isi paragraf tersebut adalah…a. FTA ASEAN –China membuat panik sejumlah pengusaha sepatu kecil-menengah di Kabupaten Mojokerto. b. Para pelanggan pengusaha sepatu di Kabupaten Mojokerto lebih memilih produk sepatu impor daripada

membeli sepatu lokal. c. Sejak Januari lalu, kerugian akibat FTA ASEAN –China dirasakan para pengusaha sepatu kecil-menengan di

Kabupaten Mojokerto. d. Akibat FTA ASEAN-China kerugian para pengusaha kecil di Kabupaten Mojokerto mencapai sekitar 50%. e. Banyak pelanggan pengusaha sepatu di kabupaten Mojokerto mengehentikan perminataan pesanan karena

menunggu pelaksanaan FTA ASEAN-China. 86. Peningkatan kesejahteraan masyarakat merupakan tanggung jawab bersama antara pemerintah dan masyarakat.

Baik pria maupun wanita, hendaknya selalu meningkatkan rasa tanggung jawab mereka terhadap kesejahteraan bersama. Kader-kader wanita diharapkan juga peduli terhadap kesejahteraan masyarakat pada umumnya. Banyak kaum wanita yang dipaksa bekerja karena suami mereka terkena PHK. Untuk itu, pemerintah perlu memperhatikan nasib kaum wanita yang suami mereka terkena PHK.Ide pokok paragraf tersebut adalah…a. Peningkatan kesejahteraan masyarakat perlu dilakukan secara bersama, antara pria dan wanita. b. Kesejahteraan keluarga korban PHK menjadi tanggung jawab pemerintah. c. Kader-kader wanita diharapkan juga peduli terhadap kesejahteraan masyarakat pada umumnya. d. Pemerintah dan masyarakat bertanggung jawab terhadap kesejahteraan masyarakat.e. Demi kesejahteraan keluarga, kaum wanita yang suami mereka di- PHK terpaksa bekerja.

87. 1) Para peneliti mengemukakan bahwa area otak untuk bernyanyi berbeda dengan area otak untuk berbicara. 2) Mereka menyajikan temuan mereka terebut pada pertemuan American Association for the Advancement of Science di San Diego. 3) Jika pusat bicara seseorang rusak oleh stroke, ia dapat belajar menggunakan pusat bernyanyinya. 4) Dalam uji coba klinis, para peneliti menunjukkan cara otak merespon terapi intonasi melodi. 5) Gottried Schlaug, profesor neurologi Beth Israel Deaconess Medical Center dan Harvard Medical School di Boston, Amerika Serikat memimpin uji coba terebut. 6) Penelitian itu menemukan bahwa pasien stroke dengan kerusakan otak membuat pasien tidak bisa bicara, tetapi bisa bernyanyi. Kalimat utama paragraf tersebut terdapat pada kalimat nomor…a. 1 b. 3 c.4 d. 5 e. 6

88. Affandi adalah putra R. Koesoema, seorang mantri ukur di pabrik gula Cileduk, Cirebon. Affandi memiliki pendidikan formal yang cukup tinggi. Bagi orang-orang segenerasinya, hanya segelintir anak negeri saja yang memperoleh pendidikan HIS, MULO, dan AMS. Akan tetapi, bakat seni lukisnya yang sangat kuat mengalahkan disiplin ilmu lain dalam kehidupannya. Bakat itu menjadikan nama Affandi tenar, sama dengan nama-nama tokoh atau pemuka bidang lainnya. Sebelum menjadi pelukis, Affandi pernah menjadi guru, tukang sobek karcis, dan pembuat gambar reklame bioskop di salah satu gedung bioskop di Bandung. Pekerjaan tersebut tidak lama dia geluti karena Affandi lebih tertarik pada seni lukis. Sekitar tahun 30-an, Affandi bergabung dalam Kelompok Lima Bandung, yaitu kelompok lima pelukis Bandung. Kelompok ini memilki andil besar dalam perkembangan seni rupa di Indonesia. Pernyataan berikut yang sesuai dengan isi teks tersebut adalah… a. Pendidikan formal Affandi (di HIS, MULO, dan AMS) cukup tinggi pada masa generasinya. b. Kelompok Lima Bandung didirikan Affandi untuk mengembangkan seni rupa Indonesia. c. Melukis menjadi pekerjaan paling lama digeluti Affandi dalam hidupnya. d. Profesi Affandi dirintis sejak awal memulai pendidikan formalnya di HIS, MULO, dan AMS. e. Affandi menjadi pelukis bersama pelukis terkenal, yaitu R. Koesoema.

89. Dalam ilmu ekonomi, inflasi adalah suatu proses meningkatnya harga-harga secara umum dan terus-menerus berkaitan dengan mekanisme paasar. Istilah inflasi juga berarti peningkatan persediaan uang yang menyebabkan kenaikan harga. Inflasi terjadi jika proses kenaikan harga berlangsung secara terus-menerus dan saling memengaruhi. Kalimat penutup yang sesuai untuk paragraf tersebut adalah…a. Dengan demikian, ada dua cara untuk mengukur tingkat inflasi, yaitu CPI dan GDP deflator. b. Akibatnya, untuk mengukur tingkat inflasi diperlukan dua cara, yaitu CPI dan GDP deflator. c. Jadi, ada dua cara untuk mengukur tingkat inflasi, yaitu CPI dan GDP deflator.

Page 32: SBMPTN REVISI

d. Dalam hal ini, ada dua cara untuk mengukur tingkat inflasi, yaitu CPI dan GDP deflator.e. Oleh karena itu, ada dua cara untuk mengukur tingkat inflasi, yaitu CPI dan GDP deflator.

90. Dulu eyang-eyang kita merawat gigi dengan cara menginang. Penelitian mutakhir membuktikan bahwa cara ini memang efektif membuat bakteri pathogen hengkang. Berdasarkan fakta inilah, minyak asirinya mengandung avikol, kavibetol, karvakrol, dan eugenol. Dibandingkan dengan venol, kandungan fitokimia sirih ini lima kali lebih kuat menghalau bakteri mulut, seperti sterptococcus mutans, streptococcus viridans, staphy lococcus aureus. Kuman-kuman itulah biang penyebab gigi berlubang. Mereka membusukkan sisa-sisa makanan yang menempel di sela-sela gigi, menghasilkan asam yang menurunkan pH cairan di sekitar gigi.Generalisasi yang tepat untuk melengkapi paragraf tersebut adalah…a. Eyang kita merawat gigi dengan cara menginang. b. Keampuhan sirih bukan sekadar mitos. c. Sirih penghalau bakteri mulut. d. Kuman-kuman penyebab gigi berlubang. e. Sisa-sisa makanan menurunkan pH cairan di sekitar gigi.

91. Sebanyak 250 tentara Jerman dikerahkan ke negara bagian Becklenberg-Vorpormmem untuk membersihkan bangaki-bangkai burung. Dalam operasi tersebut, para tentara juga dibantu para pejabat militer di tingkat negara bagian. Dengan menggunakan pesawat-pesawat tempur Tornado, mereka mencari bangkai-bangkai burung sepanjang pesisir Baltik. Pengerahan militer ini dilakukan setelah media massa Jerman berhari-hari mengkritik kerja otoritas setempat lamban. Dalam operasi tersebut ditemukan bahwa jumlah burung yang mati karena terkena flu burung di wilayah tersebut meningkat menjadi 79 ekor. Kalimat sumbang yang tidak mendukung pokok pikiran paragraf tersebut adalah kalimat….a. pertama b. kedua c. ketiga d. ketempat e. kelima

92. Memang, pasar beras telah banyak berubah selama sepuluh tahun terakhir. Namun, pasar gabah pada musim panen raya belum bekerja seperti yang diharapkan. Pada musim raya selama empat bulan, pasokan gabah begitu besar (sekitar 50% luas panen). Petani yang jumlahnya 11,3 juta orang harus menjual gabah segera untuk berbagai keperluan. Alam juga tidak bersahabat, terlalu basah karena hujan sehingga dapat menurunkan kualitas , sedangkan pihak penggilingan swasta, 95% di antaranya masih mengandalkan tua dan kecil-kecil serta kapasitas gudang terbatas. Akibatnya, kelebihan produksi gabah tidak tertampung sehingga jatuhnya harga pasti terjadi. Pernyataan berikut yang tidak sesuai dengan isi teks tersebut adalah….a. Pasar beras senantiasa berubah dalam sepuluh terakhir. b. Petani hidup dari menjual gabah. c. Alam dapat memengaruhi kualitas beras. d. Kelebihan produksi gabah dapat menurunkan harga gabah. e. Pasar gabah pada musim panen raya belum seperti yang diharapkan.

93. Banyak ahli mengatakan bahwa mengajak anak berdiskusi dalam segala hal merupakan salah satu cara membuka jalur komunikasi antara anak dan orang tua. Dengan demikian, anak merasa diterima dan merasa nyaman jka mereka ingin berbicara kepada orang tuanya. Membahas peraturan yang berlaku di rumah, misalnya, patut disertai alasan yang jelas agar anak lebih mudah menerimanya. Mendengrakan alasan dan penjelasan mereka jika salah satu aturan tidak diterima Juga merupakan suatu cara yang baik. Cara ini dapat mengembangkan pola berpikir yang juga membantu proses kedewasaan mereka. Ikhtisar dari kutipan tersebut adalah…a. Berdiskusi dengan anak-anak dan mendengarkan mereka berbicara akan membantu mengembangkan cara

berpikir dan pendewasaan anak-anak. b. Jalur komunikasi antara orang tua dan anak harus ditingkatkan sehingga anak merasa diterima dan nyaman

apabila mereka berbicara dengan orang tua. c. Saran para ahli perlu diikuti karena dapat mendukung pengembangan pola berpikir dan proses mendewasakan

anak. d. Perasaan nyaman merupakan sarana penting dalam pengembangan komunikasi antara anak dengan orang tua. e. Komunikasi yang baik adalah kunci keberhasilan hubungan antara anak dan orang tua.

94. Departemen Agama seharusnya snggup menyelenggarakan ibadah haji secara baik. Sesuai dengan undang-undang terbaru, departemen ini tetap menjadi regulator sekaligus operator, bahkan ikut menentukan anggota komisi pengawas. Peningkatan kualitas pelayanan diharapkan terjadi lewat pem- benahan manajemen. Keluhan mulai muncul ketika belum ada perbaikan yang berarti. Lihatlah, baru-baru ini 89 anggota jemaah calon haji dari Jakarta dilaporkan mengalami diare saat berada di Madinah. Penderitaan mereka diduga akibat makanan yang basi atau tidak memenuhi standar kesehatan. Insiden seperti ini tidak perlu terjadi seandainya perusahaan katering menjaga kualitas makanan yang disajikan. Kualitas pelayanan akan terjamin lagi jika tim kesehatan selalu mengecek dan mengawasi makanan untuk jemaah calon haji. ( Tempo, 211108). Pernyataan berikut merupakan opini dari teks di atas, kecuali…f. Jemaah calon haji Indonesia tidak memperoleh layanan maksimal dari Departemen Agama. g. Departemen Agama tidak bekerja secara profesional dalam penyelenggaraan ibadah haji. h. Manajemen Departemen Agama lemah sehingga tidak mampu meningkatkan pelayanan penyelenggaraan

ibadah haji. i. Tim kesehatan tidak mengecek dan mengawasi makanan sehingga jemaah calon haji makan nasi basi. j. Koordinasi tidak berjalan baik di antara komponen panitia penyelenggara ibadah haji di Departemen Agama.

Page 33: SBMPTN REVISI

IV TES POTENSI AKADEMIKA. Kemampuan Verbal

86. aliran: . . . = . . . : bersihd. air - udara d. mampat- kotorane. darah- segar e. sungai- lingkunganf. saluran- bening

87. menghargai: . . . = … : kebenciane. hak milik – musuh d. toleransi – menghujatf. sesama- kekesalan e. menghormati- marahg. menjaga- menghina

88. penuh: . . .= . . . : berhentid. isi- gerakan d. meluap- melambate. kosong- diam e. menggenang- mengeremf. banjir- pindah

89. remaja: . . .= . . .: pagid. jiwa- hari d. puber- waktue. usia- sinar e. penampilan- sarapanf. dewasa- fajar

90. mirip: . . .= . . .: laki- lakid. wajah- peran d. tampilan- pemimpine. serupa- gender e. berbeda- jenis kelaminf. kembar- maskulin

91. indah: . . . = . . .: tajamd. seni- sakit d. taman- bahayae. lukisan- runcing e. kesenangan- lukaf. perhiasan- pisau

92. tumbuh: … = . . . layud. hidup- mati d. puber- waktue. besar- lemah e. berkembang- meranggasf. subur- gugur

93. gembira: . . .= . . . : galaud. ria- berduka d. tertawa- kacaue. hati- merana e. bahagia- menangisf. histeria- gelisah

94. pengetahuan: . . .= . . .: biayad. ilmu- ongkos d. cerdas- mahale. bodoh- gratis e. sekolah- pasarf. dungu- harga

95. laut: . . .= . . .: kabupatend. pulau- peta d. nelayan- bupatie. ikan- daerah e. samudra- provinsif. air- wilayah

96. karet: . . .= . . .: populerd. lentur- aktor d. ban- terkenale. air- pendatang e. kenyal- tersohorf. kenyal- selebriti

97. …: tenteram= pengawasan: . . .d. aturan- ketat d. damai- pemeriksaane. penjagaan- kontrol e. sejahtera- pengendalianf. keamanan- teratur

98. …:ceroboh= tepat: . . .d. lali- akurat d. hati- hati – melesete. lengah- efektif e. waspada- tepat sasaranf. kecelakaan- efisien

99. ….: gila = lembab: . . .d. otak- kulit d. akal- keringe. aksi- basah e. saraf- ruangf. orang – udara

100. … : mencoblos= terbang: . . .a. bilik- angkasa d. memilih- mendaratb. gambar- pesawat e. kartu- layang- layangc. kandidat- burung

101. air: . . . = hampa: . . .a. sungai- ruang d. es- kosongb. kering- udara e. tetesan- vakum

Page 34: SBMPTN REVISI

c.dingin- pengap102. oven: . . .= . . . : foto

a. roti- kamera d. gosong- modelb. pemanggang- album e. matang- fotografer

c.panas- lensa103. murka: . . . = . . .: benturan

a. sedih- sakit d. cemberut- tendanganb. mencaci- luka e. kecewa- dorongan

c.marah- tabrakan104. buas: . . .= . . . : jet tempur

a. harimau- udara d. hiu- terbangb. serigala- manuver e. kecewa- dorongan

c. ular- laju105. hijau: . . . = . . . : laut

a. lumut- ikan d. daun- asinb. klorofil- ombak e. hutan- kapal

c.rumput- pantai106. gurun: . . . = . . .: hutan

a. pasir- belantara d. matahari- rimbunb. sahara- pohon e. kering- lebat

c.oasis- sungai107. produksi : . . . =. . . :perjalanan

a. pabrik- arah d. pekerja- kecepatanb. produk- tujuan e. upah- pulang

c.mesin- kendaraan108. buku: . . . = gawang: . . .

a. perpustakaan- lapangan bola d. ilmu- penjaga gawangb. bacaan- sepak bola e. rak- jaring

c.tulisan- pemain bola109. gerakan: . . . = kotoran: . . .

a. diam- sapu d. bergetar- berbaub. dinamis- limbah e. berpindah- pupuk

c.putaran- tercemar110. hari: … = titik: . . .

a. bulan- tanda baca d. minggu- garisb. tahun- koma e. september- kecil

c.kalender- menulis111. …: terang= suara: . . .

a. mata- bisu d. hitam- sunyib. warna- getaran e. cahaya- nyaring

c.kelam- ramai112. . …: petani= peluit: . . .

a. cangkul- wasit d. sawah- jurur parkirb. tanah- masinis e. bajak- sepak bola

c.padi- kereta api113. …: mobil= angin: . . .

a. kendaraan- udara d. jalan- baling- balingb. sedan- dingin e. rumah- butah. bensin- kincir

114. …: tuna wisma = penglihatana. harta- mata d. penghasilan- inderab. jalanan- kornea e. rumah- buta

c.spengangguran- katarak115. …: kelulusan= penghargaan: . . .

a. ujian- lomba d. ijazah- tanda jasab. siswa- pahlawan e. penilaian- hadiahc. tamat- medali

116. Gemetar berhubungan dengan . . ., sebagaimana . . . berhubungan dengan lucud. Lapar- geli d. takut- tertawae. Sakit- lelucon e. bahaya- pelawakf. Gugup- parodi

117. Merdeka berhubungan dengan . . ., sebagaimana . . . berhubungan dengan penjarad. Pahlawan- narapidana d. bebas- jeruji besie. Proklamasi- sidang e. berontak- hubunganf. Penjajahan- kebebasan

118. Redup berhubungan dengan . . ., sebagaimana . . . berhubungan dengan teriak

Page 35: SBMPTN REVISI

d. Cahaya- jauh d. seram- suarae. Terang- bisik e. lampu- memanggilf. Gelap- terkejut

119. Senja berhubungan dengan . . ., sebagaimana . . . berhubungan dengan berlarid. Malam- berjalan d. siang- kejare. Petang- kejar e. bulan- berdirif. Waktu- kejar

120. Matematika berhubungan dengan . . ., sebagaimana . . . berhubungan dengan mobild. Matriks- transportasi d. angka- kendaraane. Rumus- jalan e. hitung- mesinf. Pelajaran- sedan

121. Laptop berhubungan dengan . . ., sebagimana . . . dengan busanad. Komputer- kaos d. layar- kaose. Printer- sandal e. internet- semirf. Telepon genggam- tali

122. Ulat berhubungan dengan . . ., sebagaimana . . . berhubungan dengan berudu (kecebong)d. Bulu- kolam d. pohon- enceng gondoke. Larva- ikan e. kepompong- katakf. Kupu- kupu- katak hijau

123. Wisuda berhubungan dengan . . ., sebagaimana . . . berhubungan dengan pertunangand. Toga- cincin d. sarjana- mempelaie. Gelar- pelaminan e. kuliah- pernikahanf. Berhasil- cinta

124. Sarung tangan berhubungan dengan . . ., sebagaimana . . . berhubungan dengan bakteriologd. Steril- stetoskop d. kotor- kumuhe. Ahli forensik- mikroskop e. bersih- bakterif. Dokter- apoteker

125. Benda bersejarah berhubungan dengan . . ., sebagaimana . . . berhubungan dengan kebun binatangd. Museum- margasatwa d. peninggalan- keanekaragamane. Galeri- taman wisata e. kenangan- keindahanf. Patung- burung

126. Menguap berhubungan dengan . . ., sebagaimana . . . berhubungan dengan sakitd. Istirahat- panas badan d. tidur- istirahate. Lelah- dokter e. tempat tidur- obatf. Mengantuk- demam

127. Kaku berhubungan dengan . . ., sebagaimana . . . berhubungan dengan karetd. Tongkat-gelang d. kaki- fleksibele. Batu- lembut e. kayu- lateksf. Besi- lentur

128. Sakit berhubungan dengan . . ., sebagaimana . . . berhubungan dengan makand. Dokter- sakit d. berobat- gemuke. Makan- lemah e. pasien- gizif. Istirahat- makan

129. Marah berhubungan dengan . . ., sebagaimana . . . berhubungan dengan ledakand. Kerusakan- peledak d. tidak puas- terore. Amuk- letupan e. hawa nafsu- panasf. Emosi- bom

130. Lapangan bola berhubungan dengan . . ., sebagaimana . . . berhubungan dengan dekorasid. Tribun- bunga d. wasit- tukang fotoe. Pemain bola- seniman e. tiang gawang- panggungf. Bersepatu- bercahaya

131. Bayi berhubungan dengan . . ., sebagaimana . . . berhubungan dengan dewasad. Berjalan- anak d. elajar- mudae. Lemah- kuat e. merangkak- pemudaf. Anak- remaja

132. Vonis berhubungan dengan . . ., sebagaimana . . . berhubungan dengan pertandingand. Tuntutan- juara d. penjara- pialae. Peradilan- hasil e. hakim- suporterf. Pengacara- wasit

133. Toko berhubungan dengan . . ., sebagaimana . . . berhubungan dengan ikand. Kasir- akuarium d. makanan- pancinge. Pembeli- cacing e. pakaian- bakarf. Supermarket- paus

134. Bunyi alarm berhubungan dengan . . ., sebagaimana . . . berhubungan dengan apid. Sinyal- air d. kebakaran- arang

Page 36: SBMPTN REVISI

e. Peringatan- padam e. bahaya- asapf. Suara- abu

135. Peluit berhubungan dengan . . ., sebagaimana . . . berhubungan dengan prajuritd. Wasit- senjata d. permainan- perange. Polisi –markas e. kereta- pasukanf. Penalti- desersi

136. Nelayan : lautd. Guru : papan tulis d. pelukis: kuase. Petani: padi e. penyanyi: panggungf. Karyawan: pegawai

137. Samudra: lautd. Benua: pulau d. negara: pantaie. Internasional: batas negara e. luas: dalamf. Kapal: nelayan

138. Kuda laut: kudad. Kucing: harimau d. cacing: belute. Banteng: sapi e. merpati: elangf. Biawak: komodo

139. Beranak: mamaliad. Bertaring: karnivora d. berjemur: reptiliae. Bertelur: unggas e. berkaki empat: herbivoraf. Berkelompok: insekta

140. Kurus: gizid. Gemuk: lemak d. pintar: belajare. Sakit: dokter e. bodoh: ilmuf. Pendek: besar

141. Gigi: kunyah = jari: …b. Lentik b. pegang c. kuku d. tangan e. manis

142. Melati: bunga= bawang: …b. Umbi b. merah c. putih d. bumbu e. harum

143. … berhubungan dengan padi, sebagaimana buah berhubungan dengan . . .d. Kuning- merah d. sawah- vitamine. Makanan pokok- minuman pokok e. beras- bijif. Petani- pedagang

144. … berhubungan dengan buruh, sebagaimana supervisor berhubungan dengan . . .d. Mandor- karyawan d. demo- kerjae. Pabrik- kantor e. upah- gajif. Kecil- besar

145. … berhubungan dengan puisi, sebagaimana pemahat berhubungan dengan . . .d. Chairil Anwar- seniman d. penyair- pematunge. Syair- batu e. suara- gerakf. Baca- seni

146. Gelap= . . .b. Kelam b. kilau c. silau d. muram e. gerah

147. Pakar= . . .b. Ahli b. kompleks c. umum d.khusus e. golongan

148. Adaptasi= . . .b. tetap b. penyesuaian c. pilihan d. perencanaan e. perubahan

149. Subsantsi= . . .b. nyata b. maka c. maksud d. inti e. isi

150. Resah= . . .b. Gulana b. lekat c. gundah d. kendala e. dilema

151. Stabil >< …b. Buruk b. insidentil c. kuat d. permanen e. labil

152. Parsial >< . . .b. Internal b. eksternal c. komunal d. komuniter e. konflik

153. Permanen >< . . .b. Kuat b. sementara c. komunal d. induk e. manual

154. Versus >< . . .b. Lawan b. mitra c. pengaruh d. cegah e. ancam

155. Parasit >< . . .b. Konfusit b. inplisit c. trombosit d. analis e. simbolis

156. Gudang : barangd. Lama: baru d. bank: deposite. Buku: perpustakaan e. bangunan- antikf. Tertutup- terbuka

Page 37: SBMPTN REVISI

157. Buruh: pabrikd. Penari: studio d. pemahat: candie. Pedagang: pasar e. petani: sungaif. Nelayan: pesisir

158. Kering: lembabd. Gelap- remang- remang d. matahari- gununge. Cahaya- terang e. siang- malamf. Panas- basah

159. Paus: hiud. Baiwak: komodo d. singa: harimaue. Belut: cacing e. kambing: sapif. Merpati: elang

160. Memberi: dermawand. Mencuri: polisi d. mencuci: bajue. Melukis: kanvas e. menulis: sekretarisf. Membantu- orang

161. Kehausan: aird. Kemakmuran- kerja keras d. kelelahan- kerjae. Kebodohan- miskin e. kelaparan- panganf. Kegembiraan- hiburan

162. Terbang berhubungan dengan . . ., sebagaimana jalan berhubungan dengan . . .d. Pesawat- jauh d. burung- kupu- kupue. Burung- unta e. pesawat- daratf. Tinggi- jauh

163. Ayah berhubungan dengan . . ., sebagaimana pohon berhubungan dengan ….d. Orang tua- daun d. anak- akare. Manusia- tumbuhan e. anak- tunasf. Laki- laki- ranting

164. Es berhubungan dengan . . ., sebagaimana gula berhubungan dengan . . . .d. Padat- bubuk d. dingin- manise. Cair- kristal e. beku- kristalf. Cair- padat

165. Mobil berhubungan dengan . . ., sebagaimana kompor berhubungan dengan . . . .d. Bahan bakar- laut d. solar- kayue. Kendaraan- dapur e. busi-sumbuf. Bensin- gas

166. Pergi berhubungan dengan . . ., sebagaimana begadang berhubungan dengan. . .d. Jauh- malam d. pulang- malame. Keluar- ronda e. jauh- tidurf. Pulang- tidur

167. …: komputer = . . .: manusiad. Mouse- tangan d. CPU- tubuhe. Monitor- mata e. keyboard- jarif. Prosesor- otak

168. . . .: mobil= . . .: sampand. Bensin- dayung d. darat- aire. Kendaraan- perahu e. kendaraan- angkutanf. Darat- laut

169. . . .: pergi= . . .: awald. Datang- akhir d. pulang- zamane. Pulang- belakang e. awal- mukaf. Berangkat- kedatangan

170. . . .: makan= . . .: obatd. Nasi- resep d. nasi- lukae. Lapar- minum e. restoran- apotekf. Lapar- sakit

B.Pemahaman Wacana Teks I

Toni memiliki sebuah rak buku kecil yang terbagi atas lima tingkat, tingkat 1,2,3,4, dan 5. Tingkat I adalah yang teratas dan tingkat 5 adalah yang terbawah. Toni menyimpan buku-buku pelajaran ke dalam rak dengan aturan sebagai berikut:

d. Buku pelajaran geografi disimpan tepat di atas buku pelajaran matematika, sedangkan buku pelajaran bahasa Indonesia diletakkan tepat di bawah buku matematika.

e. Buku pelajaran ekonomi diletakkan di atas buku pelajaran fisika, sedangkan buku pelajaran fisika dan kimia berada di atas buku pelajaran geografi.

Page 38: SBMPTN REVISI

f. Buku pelajaran kimia dan biologi diletakkan saling terpisah sejauh mungkin.Tabel untuk jawaban nomor 1- 6

ekonomi, kimiafisika geografimatematikabahasa Indonesia, biologi

1. Buku yang disimpan tidak berdampingan dengan buku yang lain adalah . . . .f. Fisika, matematika, geografig. Kimia, fisika, biologih. Matematika, bahasa Indonesia, ekonomii. Bahasa Indonesia, biologi, geografij. Ekonomi, kimia, fisika

2. Adik Toni mengambil buku geografi dan menyimpannya kembali di samping buku bahasa Indonesia. tingkatan yang kosong adalah tingkatan ke . . . .b. 1 b. 2 c.3 d.4 e. 5

3. Buku yang terletak di tingkat 1 adalah buku . . . .b. Biologi b. fisika c. geografi d. kimia e. matematika

4. Buku yang letaknya setingkat dengan buku bahasa Indonesia adalah buku . . . .b. Biologi b. ekonomi c. fisika d. geografi e. matematika

5. Buku yang disimpan di tingkat ketiga adalah buku . . . .b. Biologi b. ekonomi c. fisika d. geografi e. matematika

6. Urutan lima buku dari tingkat tertinggi ke tingkat terendah adalah . . . .f. Kimia, fisika, geografi, matematika, biologig. Bahasa Indonesia, biologi, geografi, kimiah. Ekonomi, fisika, matematika, geografi, biologii. Fisika, matematika, geografi, bahasa Indonesiaj. Matematika, geografi, bahasa Indonesia, biologi, fisika

Teks IIA,B,C,D, dan E adalah lima pekerja yang mencalonkan diri menjadi ketua serikat kerja. Ada tiga program yang diajukan para calon, yaitu (1) program peningkatan keterampilan pekerja oleh A,B, dan C. (2) program olahraga oleh B,C, dan D (3) program peningkatan kesejahteraan oleh D,E, dan A. Setiap warga hanya diperkenankan memilih satu calon berdasarkan progamnya. Hasil pemungutan suara adalah sebagai berikut:

e. Enam puluh pekerja memilih program peningkatan keterampilan pekerjaf. Empat puluh lima pekerja memilih program olahragag. Sembilan puluh pekerja memilih program peningkatan kesejahteraanh. Jumlah suara terbagi sama besar pada calon- calon yang memiliki program sama

Tabel untuk jawaban nomor 7- 114) Program peningkatan keterampilan pekerja5) Program olahraga6) Program peningkatan kesejahteraan

Calon 1 2 3 Jumlah SuaraA 20 - 30 50B 20 15 - 35C 20 15 - 35D - 15 30 45E - - 30 30

Jumlah 60 45 907. Calon yang memperoleh suara terbanyak adalah . . . .

b. A b. B c. C d. D e. E8. Calon yang memperoleh suara paling sedikit adalah . . .

b. A b B c. C d. D e. E9. A dan B memilik perbedaan perolehan suara sebesar . . . .

b. 5 b. 10 c. 15 d. 20 e. 2510. Urutan dengan perolehan tiga suara terbanyak, dinulai dari yang terbanyak adalah . . . .

b. ADB b. ABC c.CAB d. DBE e. ECA11. Jika ketua terpilih mengangkat ketua seksi olahraga dari calon yang memperoleh suara terbanyak dan mengajukan

program olahraga, yang menjadi ketua seksi olahraga adalah . . . /b. A b. B c. C d. D e. E

Teks IIITono akan ke luar negeri selama satu bulan untuk urusan usaha dagangnya sehingga ia harus memilih salah satu dari tiga bawahannya yang akan menggantikannya di kantor. Beberapa informasi yang menjadi dasar keputusan Tono sebagai berikut:

Page 39: SBMPTN REVISI

d. Anto cukup kreatif dan cukup baik memimpin, namun sering sakit sehingga terkesan kurang rajin bekerja.e. Hasan paling kreatif dan cukup sehat, tetapi masih kalah dari Rudi dalam hal kerajinan dan masih kalah

dari Anto dalam kepemimpinan.f. Dalam kreativitas, Rudi masih di bawah Hasan dan Anto, tetapi ia paling rajin, paling sehat, dan paling

bagus memimpin.12. Jika Tono mempertimbangkan aspek kesehatan dan kerajinan sebagai kriteria utama dalam memilih wakilnya,

pernyataan yang paling tepat adalah . . .d. Anto berpeluang besar. d. Anto dan Rudi berpeluang sama besar.e. Hasan berpeluang besar. e. Anto, Hasan, Rudi berpeluang sama.f. Rudi berpeluang paling besar.

13. Pernyataan yang tidak tepat adalah . . . .f. Anto dan Hasan masih kalah sehat dibandingkan Rudi.g. Anto dan Rudi mengalahkan Hasan dalam kepemimpinan.h. Kekuarangan Anto dibandingkan Hasan dalam hal kreativitas.i. Kekurangan Anto dibandingkan Hasan dan Rudi adalah dalam hal kerajinan dan kesehatan.j. Anto lebih rajin daripada hasan dan lebih abik dalam memimpin.

14. Berdasarkan keunggulan – keunggulan aspeknya, calon pengganti Tono yang berpeluang paling besar secara berurutan adalah . . . .d. Anton, Rudi, Hasan d. Anton, Hasan, Rudie. Rudi, Anton, Hasan e. Rudi, Hasan, Antof. Hasan, Rudi, Anto

15. Jika Tono mempertimbangkan aspek kreativitas dan kepemimpinan sebagaikriteria utama, pernyataan yang paling tepat adalah . . . .f. Anto berpeluang terbesar mewakilinyag. Hasan berpeluang terbesar mewakilinyah. Rudi berpeluang terbesar mewakilinyai. Anto dan Rudi berpeluang sama besar mewakilinyaj. Anto, Hasan, san Rudi berpeluang sama besar mewakilinya

Teks IVSembilan murid pria dan wanita A,B,C,D,E,F,G,H dan I secara bersama membentuk barisan. Baris depan diisi oleh A,B, dan C. Baris tengah diisi oleh D,E, dan F, sedangkan baris belakangdiisi oleh G, H, dan I. Sisi kiri barisan diisi oleh B,F, dan I,sedangkan sisi kanan diisi oleh A,E, dan H. Kecuali pada barisan- barisan tersebut bercirikan sebagai berikut:

c. Murid pria dan wanita berselang- selingd. Pada barisan yang beranggotakan A, barisan terdiri atas dua muria wanita dan satu murid pria.

16. Setiap barisan diagonal terdiri atas . . . .d. Dua wanita, satu pria d. tiga priae. Dua pria, satu wanita e. tiga wanitaf. Tiga pria atau tiga wanita

17. Murid-murid yang ada di tengah barisan urut dari depan ke belakang adalah . . . b. C,G,D b.C,D,G c.D,C,G d. E,D,F e. F,D,E

18. Berapakah murid wanita di antara sembilan murid tersebut?b. 2 b. 3 c. 4 d. 5 e. 6

19. Jenis kelamin B, E, dan I adalah . . . .d. B wanita, E pria, I wanita d. B wanita, I pria, E wanitae. E wanita, B pria, I wanita e.E wanita, B wanita, I priaf. I wanita, B pria, E pria

Teks VDalam pertandingan bulu tangkis Arman selalu kalah melawan Bambang, tetapi dalam cabang olahraga yang lainnya ia selalu menang bila melawan Bambang. Candra selalu menang dalam pertandingan tenis meja melawan Bambang, tetapi dalam cabang bulu tangkis ia akan kalah bila bertanding melawan Arman. Dudi adalah pemain bulu tangkis terbaik, tetapi dalam cabang tenis meja dia tidak sebaik Bambang. Dalam cabang tenis meja, Edi lebih baik daripada Arman, sedangkan dalam cabang bulu tangkis ia menempati urutan tepat di bawah Dudi.

20. Siapakah pemain tenis meja terbaik di antara kelima atlet tersebut?b. Arman b. Bambang c. Candra d. Dudi e. Edi

21. Untuk cabang olahraga tenis meja, ranking pertama terbaik yang manakah yang paling tepat dari urutan di bawah ini?d. Bambang- Arman- Candra- Dudi- Edi d. Edi- Dudi- Candra- Bambang- Armane. Arman- Bambang- Candra- Edi- Dudi e. Edi- Arman- Candra- Bambang- Dudi f. Dudi- Edi- Candra- Bambang- Arman

22. Untuk cabang olahraga bulu tangkis, ranking pemain terbaik manakah yang paling tepat dari urutan di bawah ini?d. Dudi- Edi- Arman- Bambang- Candra d. Bambang- Dudi- Edi- Arman- Candrae. Bambang- Arman- Dudi- Edi- Candra e. Dudi- Edi- Candra- Bambang- Armanf. Dudi- Edi- bambang- Arman- candra

Teks VIMinggu ini yang mendapat giliran piket kelas adalah tiga anak wanita, yaitu Ita, Ayu, dan Lani, dan dua anak pria,

Page 40: SBMPTN REVISI

yaitu Putra dan Dani. Setiap hari harus ada tiga orang yang membersihkan kelas mulai dari Senin sampai Jumat dengan ju,lah yang sama. Pengaturan jadwal harus mempertimbangkan hal-hal berikut:

e. Setiap Jumat Ita dan Dani ikut kegiatan pramuka sehingga tidak dapat membersihkan kelas.f. Setiap Senin dan Rabu Ayu harus segera pulang untuk menjemput adiknya di TK.g. Setiap hari harus ada anak pria yang membersihkan kelas.h. Lani harus segera pergi ke tempat les matematika setiap Senin dan Kamis.

23. Siswa yang membersihkan kelas pada tiap Selasa adalah . . . .d. Dani, lani, dan Ita d. Lani, Ayu, dan Danie. Putra, Ita, dan Dani e. Ita, Dani, dan Putraf. Lani, Ita, dan Putra

24. Ita dan Dani mendapat giliran bekerja membersihkan kelas setiap . . . .d. Selasa dan Kamis d. rabu dan Jumate. Senin dan Selasa e. Rabu dan Kamisf. Senin dan Kamis

25. Dani dan Putra tidak pernah mendapat giliran meambersihkan bersama- sama, kecuali pada . . . .b. Senin b. Selasa c. Rabu d. Kamis e. Jumat

Teks VIISetiap berangkat ke sekolah Nina membawa bekal makanan. Bekal makanan yang dibawa Nina diatur oleh ibunya tanpa memperhitungkan hari ketika Nina tidak masuk sekolah. Nasi goreng dan nasi putih dibawa Nina secara bergantian setiap sekolah. Lauk untuk dua hari sekolah berurutan berupa ayam goreng baru kemudian telur goreng untuk satu hari sekolah berikutnya, dan seterusnya berulang.

e. Hari pertama pada minggu pertama tahun ajaran adalah Senin.f. Bekal pertama kali yang dibawa Nina berupa nasi goreng dan ayam goreng.g. Nina tidak masuk sekolah pada Selasa dan Sabtu minggu kedua karena sakit.h. Rabu minggu kelima adalah hari libur besar nasional.

Tabel jadwal membawa bekal (tabel yang sama untuk empat minggu berikutnya)Senin nasi goreng dan ayam gorengSelasa nasi putih dan ayam gorengRabu nasi goreng dan telur gorengKamis nasi putih dan ayam gorengJumat nasi goreng dan ayam gorengSabtu nasi putih dan telur gorengMinggu nasi goreng dan ayam goreng

26. Pada hari apakah, selama lima minggu pertama Nina dibekali dengan nasi goreng dan ayam goreng?b. Senin b. Selasa c. Rabu d. Kamis e. Jumat

27. Pada hari apakah selama lima minggu pertama Nina lebih sering membawa bekal nasi putih dan telur goreng?b. Senin b. Selasa c. Rabu d. Kamis e.Jumat

28. Pada hari apa sajakah, pada minggu ketiga, Nina dibekali nasi putih dan ayam goreng?d. Senin dan Rabu d. Kamis dan Sabtue. Selasa dan Kamis e. Jumat dan Seninf. Rabu dan Jumat

29. Selama lima minggu pertama berpa kalikah Nina membawa bekal nasi goreng dan telur goreng?b. 8 b. 7 c. 6 d. 5 e. 4

30. Bacalah dengan cermat teks berikut!Seorang karyawan mengatur enam ruang kerja untuk enam staf dengan urutan ruang nomor 1 sampai 6 dengan aturan sebagai berikut!7. Bu Rati sering bercakap- cakap yang suaranya terdengar keras ke ruang sebelahnya.8. Pak Mara dan Pak Bono ingin berdekatan agar dapat berkoordinasi. 9. Bu Heni meminta ruang nomor 5 yang berjendela lebar.10. Pak Dedi tidak suka pekerjaannya terganggu oleh suara- suara. 11. Pak Tasman, Pak mara, dan pak Dedi adalah perokok.12. Bu Heni alergi dengan asap rokok.Dari keterangan yang diperoleh, pengaturan ruangan yang tepat adalah sebagai berikut:Dedi Tasman Mara Bono Heni RatiTiga karyawan perokok seharusnya ditempatkan di ruang nomor . . . .d. 1,2, dan 4 d. 2,3, dan 4e. 2,3, dan 6 e. 1,2, dan 6f. 1,2, dan 3

C.KEMAMPUAN PENALARAN 1. Semua sarung bermotif kotak- kotak.

Sebagian kebaya adalah sarunga. Semua sarung adalah kebaya.b. Semua kebaya bermotif kotak- kotak.c. Sebagian kebaya bermotif kotak- kotak.

Page 41: SBMPTN REVISI

d. Sebagian yang bermotif kotak- kotak adalah kebaya.e. Sebagian yang bermtif kotak- kotak adalah sarung.

2. Semua musik memiliki tempo cepat.